You are on page 1of 124

www.bookspar.

com | VTU NOTES | QUESTION PAPERS | NEWS | RESULTS | FORUMS

Engineering Mathematics – I
(10 MAT11)

LECTURE NOTES
(FOR I SEMESTER B E OF VTU)

www.bookspar.com

www.bookspar.com | VTU NOTES | QUESTION PAPERS 1 of 124


www.bookspar.com | VTU NOTES | QUESTION PAPERS | NEWS | RESULTS | FORUMS

2011

ENGNEERING MATHEMATICS – I

Content

CHAPTER

UNIT I DIFFERENTIAL CALCULUS – I

UNIT II DIFFERENTIAL CALCULUS – II

UNIT III DIFFERENTIAL CALCULUS – III

www.bookspar.com | VTU NOTES | QUESTION PAPERS 2 of 124


www.bookspar.com | VTU NOTES | QUESTION PAPERS | NEWS | RESULTS | FORUMS

2011

UNIT - I
DIFFERENTIAL CALCULUS – I
Introduction:
The mathematical study of change like motion, growth or decay is calculus. The Rate of change of
given function is derivative or differential.

The concept of derivative is essential in day to day life. Also applicable in Engineering, Science,
Economics, Medicine etc.

Successive Differentiation:
Let y = f (x) --(1) be a real valued function.
dy
The first order derivative of y denoted by or y’ or y1 or ∆1
dx 2
The Second order derivative of y denoted by d yor y’’ or y2 or ∆2
dx 2
Similarly differentiating the function (1) n-times, successively,
dny
the n order derivative of y exists denoted by n or yn or yn or ∆n
th
dx
The process of finding 2nd and higher order derivatives is known as Successive Differentiation.

nth derivative of some standard functions:


1. y = eax
Sol : y1 = a eax
y2 = a2 eax
Differentiating Successively

yn = an eax
ie. Dn[eax] = an eax
For, a =1 Dn[ex] = ex

www.bookspar.com | VTU NOTES | QUESTION PAPERS 3 of 124


www.bookspar.com | VTU NOTES | QUESTION PAPERS | NEWS | RESULTS | FORUMS

www.bookspar.com | VTU NOTES | QUESTION PAPERS 4 of 124


www.bookspar.com | VTU NOTES | QUESTION PAPERS | NEWS | RESULTS | FORUMS

2011

www.bookspar.com | VTU NOTES | QUESTION PAPERS 5 of 124


www.bookspar.com | VTU NOTES | QUESTION PAPERS | NEWS | RESULTS | FORUMS

www.bookspar.com | VTU NOTES | QUESTION PAPERS 6 of 124


www.bookspar.com | VTU NOTES | QUESTION PAPERS | NEWS | RESULTS | FORUMS

7
www.bookspar.com | VTU NOTES | QUESTION PAPERS 7 of 124
www.bookspar.com | VTU NOTES | QUESTION PAPERS | NEWS | RESULTS | FORUMS

2011

Leibnitz’s Theorem :
It provides a useful formula for computing the nth derivative of a product of two functions.
Statement : If u and v are any two functions of x with un and vn as their nth derivative. Then the nth
derivative of uv is

(uv)n = u0vn + nC1 u1vn-1 + nC2u2vn-2 + …+nCn-1un-1v1+unv0


Note : We can interchange u & v (uv)n = (vu)n,
nC = n , nC2 = n(n-1) /2! , nC3= n(n-1)(n-2) /3! …
1

1. Find the nth derivations of eax cos(bx + c)


Solution: y1 = eax – b sin (bx +c) + a eax cos (b x + c), by product rule.
.i.e, y1 = eax [a cos (bx + c ) − b sin (bx + c )]
Let us put a = r cos θ , and b = r sin θ .
∴ a 2 + b 2 = r 2 and tan θ = b / a

.ie., r = a 2 + b 2 and θ = tan-1 (b/a)


Now, y1 = e ax [r cos θ cos(bx + c) − r sin θ sin( bx + c)]
Ie., y1 = r eax cos (θ + bx + c )
where we have used the formula cos A cos B – sin A sin B = cos (A + B)
Differentiating again and simplifying as before,
y2 = r2 eax cos (2θ + bx + c ) .
Similarly y3 = r3 e ax cos (3θ + bx + c ) .
………………………………………
Thus y n = r n e ax cos(nθ + bx + c )

Where r = a 2 + b 2 and θ = tan-1 (b/a).


Thus Dn [eax cos (b x + c)]

[ [
= ( a 2 + b 2 ) n e ax cos n tan −1 (b / a ) + bx + c ]]

www.bookspar.com | VTU NOTES | QUESTION PAPERS 8 of 124


www.bookspar.com | VTU NOTES | QUESTION PAPERS | NEWS | RESULTS | FORUMS

2. Find the nth derivative of log 4x2 + 8x + 3

Solution : Let y = log 4 x 2 + 8x + 3 = log (4x2 + 8x +3) ½


1
ie., y = log (4x2 + 8x +3) ∵ log xn = n log x
2
1
y= log { (2x + 3) (2x+1)}, by factorization.
2
1
∵ y= {log (2x + 3) + log (2x + 1)}
2

1 ⎧ (− 1) (n − 1)!2 n (− 1) (n − 1)!2 n ⎫
n −1 n −1

Now y n = ⎨ + ⎬
2 ⎩ (2x + 3)n (2x + 1)n ⎭
⎧ 1 1 ⎫
Ie., yn = 2n-1 (-1) n-1 (n-1) ! ⎨ + n ⎬
⎩ (2 x + 3) (2 x + 1) ⎭
n

3. Find the nth derivative of log 10 {(1-2x)3 (8x+1)5}


Solution : Let y = log 10 {1-2x)3 (8x+1)5}
It is important to note that we have to convert the logarithm to the base e by the property:
log e x
log10 x =
log e 10

Thus y =
1
log e 10
{
log e (1 − 2 x ) (8x + 1)
3 5
}
Ie., y =
1
{3 log(1 − 2x ) + 5 log(8x + 1)}
log e 10

1 ⎧ (− 1) (n − 1)!(− 2) (− 1)n −1 (n − 1)!8n ⎫


n −1 n
∴ yn = ⎨3. + 5 ⎬
loge 10 ⎩ (1 − 2x )n (8x + 1)n ⎭

Ie., yn =
(− 1) (n − 1)!2 n ⎧ 3(− 1)
n −1 n
+
5(4) ⎫
n

⎨ n ⎬
⎩ (1 − 2x ) (8x + 1) ⎭
n
log e 10

www.bookspar.com | VTU NOTES | QUESTION PAPERS 9 of 124


www.bookspar.com | VTU NOTES | QUESTION PAPERS | NEWS | RESULTS | FORUMS

4. Find the nth derivative of e2x cos2 x sin x


⎡1 + cos 2 x ⎤
Solution : >> let y = e2x cos2 x sin x = e2x ⎢ ⎥ sin x
⎣ 2 ⎦
e2x
ie., y = (sin x + sin x cos 2x)
2
e2x ⎧ ⎫
⎨sin x + [sin 3x + sin (− x )]⎬
1
=
2 ⎩ 2 ⎭
e2x
= (2 sin x + sin 3x − sin x )∵ sin (-x) = -sin x
4
e2x
∴y = (sin x + sin 3x)
4

Now y n = { (
1 n 2x
4
)
D e sin x + D n e 2 x sin 3x ( )}
Thus y n =
1
4
{( 5 ) en 2x
[
sin n tan −1 (1 2 ) + x + ] ( 13 ) e n 2x
[
sin n tan −1 (3 2 ) + 3x ]}

∴ yn =
e2x
4
{( 5 ) sin[n tan
n −1
(1 2) + x ]+ ( ) [
13 sin n tan −1 (3 2) + 3x
n
]}

5. Find the nth derivative of e2x cos 3x


1
Solution : Let y=e2x cos3 x = e 2x. (3 cos x + cos 3x)
4
1
Ie., y= (3 e2x cos x + e2x cos 3x)
4
1
∴ yn = {3Dn (e2x cos x) + Dn (e2x cos 3x)}
4

yn =
1
4
{( )
n
[
3 5 e 2 x cos n tan −1 (1 2 ) + x + ] ( 13 ) e n 2x
[
cos n tan −1 (3 2 ) + 3 x ]}

Thus y n =
e2x
4
{( ) [ ] ( )
3 5 cos n tan −1 (1 2 ) + x + 13 cos n tan −1 (3 2) + 3x
n n
[ ]}

www.bookspar.com | VTU NOTES | QUESTION PAPERS 10 of 124


www.bookspar.com | VTU NOTES | QUESTION PAPERS | NEWS | RESULTS | FORUMS

x2
6. Find the nth derivative of
(2x + 1)(2x + 3)
x2
Solution : y= is an improper fraction because; the degree of the
(2x + 1)(2x + 3)
numerator being 2 is equal to the degree of the denominator. Hence we must divide and
rewrite the fraction.
x2 1 4x 2
y= = . for convenience.
4 x 2 + 8x + 3 4 4 x 2 + 8x + 3

1
2
4x
4 x 2 + 8x + 3
4x2 +8x +3
− 8x − 3

1⎡ − 8x − 3 ⎤
∴ y= 1+ 2
4 ⎣ 4 x + 8x + 3 ⎥⎦

1 1 ⎡ 8x + 3 ⎤
Ie., y = −
4 4 ⎢⎣ 4 x 2 + 8x + 3 ⎥⎦
The algebraic fraction involved is a proper fraction.
1 ⎡ 8x + 3 ⎤
Now yn = 0 − Dn ⎢ 2 ⋅
4 ⎣ 4 x + 8x + 3 ⎥⎦
8x + 3 A B
Let = +
(2x + 1)(2x + 3) 2x + 1 2x + 3
Multiplying by (2x + 1) (2x + 3) we have, 8x + 3 = A (2x + 3) + B (2x + 1)
................(1)
By setting 2x + 1 = 0, 2x + 3 = 0 we get x = -1/2, x = -3/2.
Put x = -1/2 in (1): -1 -1 + A (2) ⇒ A = -1/2
Put x = -3/2 in (1): -9 = B (-2) ⇒ B = 9/2
1 ⎧ 1 ⎡ 1 ⎤ 9 n ⎡ 1 ⎤⎫
∴ y n = − ⎨− D n ⎢ + D ⎬
4 ⎩ 2 ⎣ 2x + 1⎥⎦ 2 ⎢⎣ 2x + 3 ⎥⎦ ⎭

www.bookspar.com | VTU NOTES | QUESTION PAPERS 11 of 124


www.bookspar.com | VTU NOTES | QUESTION PAPERS | NEWS | RESULTS | FORUMS

1⎧
= − ⎨(− 1) ⋅
(− 1)n n!2n + 9 ⋅ (− 1)n n!2n ⎫

8⎩ (2x + 1)n +1 (2x + 3)n +1 ⎭

ie., y n =
(− 1) n!2 n ⎧
n +1
1
+
9 ⎫
⎨ n +1 ⎬
8 ⎩ (2x + 1)
n +1
(2x + 3) ⎭

x4
7. Find the nth derivative of
( x + 1) ( x + 2)

x4
Solution : y = is an improper fraction.
( x + 1) ( x + 2)
(deg of nr. = 4 > deg. of dr. = 2)

On dividing x4 by x2 + 3 x + 2, We get
⎡ − 15 x − 14 ⎤
y = ( x2 – 3x + 7 ) + ⎢ 2 ⎥
⎣ x + 3x + 2 ⎦
⎡ 15 x − 14 ⎤
∴ yn = Dn (x2-3x+7)-Dn ⎢ 2 ⎥
⎣ x + 3x + 2 ⎦
But D = ( x2 – 3x + 7 ) = 2x – 3, D2 ( x2 – 3x + 7 ) = 2
D3( x2 – 3x + 7 ) = 0......... Dn ( x2 – 3x + 7 ) = 0 if n > 2
⎡ 15 x + 14 ⎤
Hence yn = -Dn ⎢ ⎥
⎣ ( x + 1) ( x + 2) ⎦
15 x + 14 A B
Now, let Dn = +
x + 3x + 2 ( x + 1) ( x + 2)
2

=> 15x+ 14 = A(x+2) + B(x+ 1 )


Put x = - 1 ; - 1 = A ( 1 ) or A = - 1
Put x = - 2 ; - 16 = B ( - 1 ) or B = 16
⎧ ⎡ 1 ⎤ ⎡ 1 ⎤⎫
Yn = ⎨− D n ⎢ ⎥ + 16 D n ⎢ ⎬
⎩ ⎣ x + 1⎦ ⎣ x + 2 ⎥⎦ ⎭

www.bookspar.com | VTU NOTES | QUESTION PAPERS 12 of 124


www.bookspar.com | VTU NOTES | QUESTION PAPERS | NEWS | RESULTS | FORUMS

(−1) n n ! 1n (−1) n n ! 1n
= − 16
( x + 1) n +1 ( x + 2) n +1
⎧ 1 16 ⎫
yn = (−1) n n ! ⎨ n +1
− ⎬n> 2
⎩ ( x + 1) ( x + 2) n +1 ⎭

8. Show that
d n ⎛ log x ⎞ (−1) n n! ⎧ 1 1 1⎫
n ⎜ ⎟= n +1 ⎨log x − 1 − − − ⎬
dx ⎝ x ⎠ x ⎩ 2 3 n⎭
log x 1 1
Solution : Let y = = log x. and let u = log x, v =
x x x
We have Leibnitz theorem,
(uv)n = uvn + nC1 u1v n −1 + nC2 u 2 v n − 2 + .... + u n v …… (1)

(−1) n−1 (n − 1)!


Now, u = log x ∴ un =
xn
1 ( −1) n n!
v = ∴ vn =
x x n+1
Using these in (1) by taking appropriate values for n we get,
⎛ log x ⎞ (−1) n n! 1 (−1) n−1 (n − 1)!
Dn = ⎜ ⎟ = log x.. n +1 + n .
⎝ x ⎠ x x xn

n(n − 1) ⎛ 1 ⎞ (−1) n−2 (n − 2)!


+ ⎜− ⎟
1. 2 ⎝ x 2 ⎠ x n −1
(−1) n−1 (n − 1)! 1
+ ...... + .
xn x
(−1) n n! (−1) n−1 n!
Ie.. = log x +
x n+1 x n+1
(−1) n − 2 n! (−1) n −1 (n − 1)!
− + .... +
2 x n +1 x n +1
(−1) n−2 n! ⎡ −1 (−1) −2 (−1) −1 (n − 1)!⎤
− log x(−1) − + .... +
x n+1 ⎢⎣ 2 n1 ⎥

1 1
Note : (-1)-1 = = −1; (−1) −2 = =1
−1 (−1) 2

www.bookspar.com | VTU NOTES | QUESTION PAPERS 13 of 124


www.bookspar.com | VTU NOTES | QUESTION PAPERS | NEWS | RESULTS | FORUMS

2011
(n − 1)! (n − 1)! 1
Also = =
n! n (n − 1)! n

d n ⎡ log x ⎤ ( −1) n n! ⎡ 1 1 1⎤
∴ n ⎢ ⎥ = n +1 ⎢ log x − 1 − − ... − ⎥
dx ⎣ x ⎦ x ⎣ 2 3 n⎦

9. If yn= Dn (xn logx)


Prove that yn = n yn-1+(n-1)! and hence deduce that
⎛ 1 1 1⎞
yn = n ⎜ log x + 1 + + + .... + ⎟
⎝ 2 3 n⎠
Solution : yn = Dn(xn log x) = Dn-1 {D (xn log x}
⎧ n 1 n −1 ⎫
= Dn-1 ⎨ x . + nx log x ⎬
⎩ x ⎭

= Dn-1(xn-1) + nDn-1 (xn-1 log x}


∴ yn = (n-1)! +nyn-1. This proves the first part.
Now Putting the values for n = 1, 2, 3...we get
y1 = 0! + 1 y0 = 1 + log x = 1! (log x + 1 )
y2 = l! + 2y1 = l+2 (l + log x)
⎛ 1⎞
ie., y2 = 21og x + 3 = 2(log x + 3/2) = 2! ⎜ log x + 1 + ⎟
⎝ 2⎠
y3 = 2! + 3y2 = 2 + 3(2 log x + 3)
⎛ 1 1⎞
ie., y3 = 61og x+ll = 6 (log x + ll/6) = 3! ⎜ log x + 1 + + ⎟
⎝ 2 3⎠
…………………………………………………………………………..
⎛ 1 1 1⎞
y n = n!⎜ log x + 1 + + + ... + ⎟
⎝ 2 3 n⎠

10. If y = a cos (log x) + b sin ( log x), show that


x2y2 + xy1 + y = 0. Then apply Leibnitz theorem to differentiate this result n times.
or
If y = a cos (log x) + b sin (log x ), show that
x2yn + 2 + (2n+l)xyn + l+(n2+1)yn = 0. [July-03]

www.bookspar.com | VTU NOTES | QUESTION PAPERS 14 of 124


www.bookspar.com | VTU NOTES | QUESTION PAPERS | NEWS | RESULTS | FORUMS

Solution : y = a cos (log x) + b sin (log x)


Differentiate w.r.t x
1 1
∴ y1 = -a sin (log x) + b cos (log x).
x x
(we avoid quotient rule to find y2) .
=> xy1 = - a sin (log x) + b cos (log x)
Differentiating again w.r.t x we have,
1
xy2 + 1 y1 = - a cos (log x) + b sin ( log x)
x
or x2y2 + xy1 = - [ a cos (log x) + b sin (log x) ] = -y
∴ x2y2+xy1+y = 0
Now we have to differentiate this result n times.
ie., Dn (x2y2) + Dn (xy1) + Dn (y) = 0
We have to employ Leibnitz theoreom for the first two terms.
Hence we have,
⎧ 2 n n −1 n(n − 1) ⎫
⎨ x . D ( y 2 ) + n. 2 x. D ( y 2 ) + . 2 . D n−2 ( y 2 ) )⎬
⎩ 1. 2 ⎭
{x. D n
}
( y1 ) + n. 1 . D n −1 ( y1 ) + y n = 0

ie., {x2yn + 2 + 2n x yn + 1 + n (n – 1)yn} + {xyn+1+nyn}+yn = 0


ie., x2yn + 2 + 2n x yn + 1 + n2yn - nyn + xyn+1+nyn+yn = 0
ie., x2yn + 2 + (2n+l)xy n+l + (n2+l)yn = 0

11. If cos-1 (y/b ) = log (x/n)n, then show that


x2yn + 2 + (2n+l) xy n+l + 2n2yn = 0
Solution :By data, cos-1 (y/b) = n log (x/n) ∴ log(am) = m log a
y
=> = cos [n log (x/n )]
b
or y = b . cos [ n log (x/n)]
Differentiating w.r.t x we get,

www.bookspar.com | VTU NOTES | QUESTION PAPERS 15 of 124


www.bookspar.com | VTU NOTES | QUESTION PAPERS | NEWS | RESULTS | FORUMS

1 1
y1 = -b sin [n log (x/n)] ⋅ n ⋅ ⋅
(x / n ) n
or xy1 = - n b sin [n log (x/n )]
Differentiating w.r.t x again we get,
1 1
xy2 + 1. y1 = - n . b cos [ n log (x/n )] n .
( x / n) n
or x (xy2+y1) = n2b cos [n log (x/n) ] =-n2y, by using (1).
or x2y2 +xyl + n2y = 0
Differentiating each term n times we have,
D(x2y2) + Dn(xy1) + n2Dn (y) = 0
Applying Leibnitz theorem to the product terms we have,
⎧ 2 n( n − 1) ⎫
⎨ x y n + 2 + n. 2 x. y n +1 + . 2 . yn ⎬
⎩ 1. 2 ⎭
+ {xy n +1 + n. 1 . y n }+ n y n = 0
2

ie x2yn+2 + 2 x yn+1 + n2yn + xy n+1+ nyn + n2yn=0


or x2 yn+2 + (2n + l) xyn+1 + 2n2yn = 0

12. If y = sin( log (x2 + 2 x + 1)),


or [Feb-03]
If sin-1 y = 2 log (x + 1), show that
(x+l)2yn + 2 + (2n+1)(x+1)yn+l + (n2 + 4)yn = 0
Solution : By data y = sin log (x2 + 2 x + 1 )
1
∴ y1 = cos log (x2 + 2 x + 1) 2x + 2
( x + 1) 2
1
ie., y1 = cos log (x2 + 2 x + 1) 2 (x + 1)
x + 2x +1
2

2 cos log( x 2 + 2 x + 1 )
ie., y1 =
( x + 1)
or (x + 1) y1 = 2 cos log (x2 + 2 x + 1 )
Differentiating w.r.t x again we get

www.bookspar.com | VTU NOTES | QUESTION PAPERS 16 of 124


www.bookspar.com | VTU NOTES | QUESTION PAPERS | NEWS | RESULTS | FORUMS

1
(x+1)y2 + 1 y1 = -2 sin log (x2 + 2x + 1) . 2( x + 1)
( x + 1) 2
or (x + 1)2y2 + (x+1) y1 = -4y
or (x+l )2y2 + (x+l) y1 + 4y = 0 ,
Differentiating each term n times we have,
Dn [(x + 1)2y2] +Dn [(x+ 1)y1] + Dn [y] = 0
Applying Leibnitz theorem to the product terms we have,
⎧ n(n − 1) ⎫
⎨( x + 1) y n+ 2 + n. 2( x + 1). y n+1 +
2
.2 . y n ⎬
⎩ 1. 2 ⎭
+ {(x+l) yn + 1+n. 1 .yn} + 4yn = 0
ie., (x+l)2yn + 2 + 2n (x+1)yn+1
+ n2yn-nyn + (x+l)yn+l + nyn + 4yn = 0
ie., (x+l)2yn + 2 + (2n + l) (x + l) y n+ 1 + (n2 + 4)yn = 0

( )
13. If = log x + 1 + x 2 prove that
(1 + x2) yn+2 + (2n + 1) xyn+1 + n2yn = 0

>> By data, y = log x + 1 + x 2 ( )


1 ⎧⎪ 1 ⎫⎪
∴ y1 = ⎨1 + . 2 x ⎬
( x + 1 + x 2 ) ⎪⎩ 2 1+ x 2 ⎪⎭

1 1+ x2 + x 1
Ie., y1 =
( x + 1+ x ) 2
1+ x 2
1+ x2

or 1 + x 2 y1 = 1
Differentiating w.r.t.x again we get
1
1 + x 2 y2 + .2 x. y1 = 0
2 1+ x 2 )

or (1+x2)y2 + xy1 = 0
Now Dn [(l+x2)y2] + Dn[xy1] = 0
Applying Leibnitz theorem to each term we get,

17
www.bookspar.com | VTU NOTES | QUESTION PAPERS 17 of 124
www.bookspar.com | VTU NOTES | QUESTION PAPERS | NEWS | RESULTS | FORUMS

⎧ n(n − 1) ⎫
⎨(1 + x ) y n+ 2 + n. 2 x . y n+1 +
2
.2 . y n ⎬
⎩ 1 .2 ⎭
+ [x . yn + 1+n .1 yn] = 0
Ie., (1 + x2) yn +2 + 2 n x yn + 1 + n2yn – nyn + xyn+l+ nyn = 0
or (l+x2)yn + 2 + (2n + l)xyn+1+n2yn = 0

14. If x = sin t and y = cos mt, prove that


(l-x2)yn + 2-(2n+1)xyn+l + (m2-n2)yn = 0. [Feb-04]
Solution : By data x = sin t and y = cos mt
x = sin t => t = sin-1 x and y = cos mt becomes
y = cos [ m sin-1x)
Differentiating w.r.t.x we get
m
y1 = - sin (m sin-1x)
1− x2

or 1 + x 2 y1 = - m sin (m sin-1x)
Differentiating again w.r.f .x we get,
1 m
1 − x 2 y2 + (−2 x ) y1 = − m cos (m sin −1 x ).
2 1− x 2
1− x2
or (1 -x2)y2-xyl = -m2y
or (1 -x2)y2 –xy1 +m2y = 0
Thus (1-x2)yn+2-(2n+1)xyn+1+(m2-n2)yn=0

15. If x = tan ( log y), find the value of


(l+x2)yn+1 + (2nx-l) yn+n(n-1)yn-1 [July-04]
Solution : By data x = tan(log y) => tan-1 x = log y or y = etan-1 x Since the desired relation involves
yn+1, yn and yn-1 we can find y1 and differentiate n times the result associated with y1 and y.
−1 −1 1
Consider y = e tan x
⋅ ∴ y. = e tan x

1+ x2
or (1 +x2)y1 = y
Differentiating n times we have

www.bookspar.com | VTU NOTES | QUESTION PAPERS 18 of 124


www.bookspar.com | VTU NOTES | QUESTION PAPERS | NEWS | RESULTS | FORUMS

2011
Dn[(l+x2)y1]=Dn[y]
Anplying Leibnitz theorem onto L.H.S, we have,
{(l+x2)Dn(y1) + n .2x .Dn-1 (y1)
n( n − 1)
+ .2 .D n − 2 ( y1 )} = y n
1 .2
Ie., (1+x2)yn+1+2n x yn + n (n-1) yn-1-yn=0
Or (l+x2)yn + 1 + (2nx-l)yn + n(n-l)yn-1 = 0

www.bookspar.com | VTU NOTES | QUESTION PAPERS 19 of 124


www.bookspar.com | VTU NOTES | QUESTION PAPERS | NEWS | RESULTS | FORUMS

Continuity & Differentiability

Some Fundamental Definitions

A function f (x) is defined in the interval I, then it is said to be continuous at a point x = a


if lim f ( x) = f (a)
x→a f ( x + h) − f (a)
A function f (x) is said to be differentiable at x = a if lim = f '(a) exists a ∈I
h→0 h

⎧− x −1 ≤ x ≤ 0
Ex : Consider a function f (x) is defined in the interval [-1,1] by f (x) = x =⎨
⎩ x 0 ≤ x ≤1

It is continuous at x = 0
But not differentiable at x = 0

Note : If a function f (x) is differentiable then it is continuous, but converse need not be true.
Geometrically :

(1) If f (x) is Continuous at x =a means, f (x) has no breaks or jumps at the point x = a
⎧−1 −1 ≤ x ≤ 0
Ex : f (x) = ⎨
⎩ x 0 < x ≤ 1

Is discontinuous at x=0

(2) If f (x) is differentiable at x = a means, the graph of f (x) has a unique tangent at the point or graph
is smooth at x = a

1. Give the definitions of Continuity & Differentiability:

Solution: A function f (x) is said to be continuous at x = a, if corresponding to an arbitrary positive


number ε, however small, their exists another positive number δ such that.

⏐f (x) – f (a)⏐ < ε, where ⏐x - a⏐ < δ

It is clear from the above definition that a function f (x) is continuous at a point ‘a’.

If (i) it exists at x = a

(ii) Lt f (x) = f (a)


x→a

i.e, limiting value of the function at x = a is to the value of the function at x = a

www.bookspar.com | VTU NOTES | QUESTION PAPERS 20 of 124


www.bookspar.com | VTU NOTES | QUESTION PAPERS | NEWS | RESULTS | FORUMS

Differentiability:
A function f (x) is said to be differentiable in the interval (a, b), if it is differentiable at
every point in the interval.
In Case [a,b] the function should posses derivatives at every point and at the end points a & b i.e., Rf1
(a) and Lf1 (a) exists.

2. State Rolle’s Theorem with Geometric Interpretation.


Statement: Let f (x) be a function is defined on [a,b] & it satisfies the following Conditions.

(i) f (x) is continuous in [a,b]

(ii) f (x) is differentiable in (a,b)

(iii) f (a) = f (b)


Then there exists at least a point C∈ (a,b), Here a < b such that f1 ( c ) = 0

Proof:
Geometrical Interpretation of Rolle’s Theorem:

Y y = f (x)

P R

A B A B

f(a) Q S
→f(a) → f(b)
O x=a x=c x=b
x =a c1 c2 c3 c4 x = b

Let us consider the graph of the function y = f (x) in xy – plane. A (a,.f(a)) and
B (b, f( b ) ) be the two points in the curve f (x) and a, b are the corresponding end points of A & B
respectively. Now, explained the conditions of Rolle’s theorem as follows.

(i) f (x) is continuous function in [a,b], Because from figure without breaks or jumps in
between A & B on y = f (x).

(ii) f (x) is a differentiable in (a,b), that means let us joining the points A & B, we
get a line AB.

∴ Slope of the line AB = 0 then ∃ a point C at P and also the tangent at P (or Q or R or S) is
Parallel to x –axis.

∴ Slope of the tangent at P (or Q or R or S) to be Zero even the curve y = f (x) decreases or
increases, i.e., f (x) is Constant.

www.bookspar.com | VTU NOTES | QUESTION PAPERS 21 of 124


www.bookspar.com | VTU NOTES | QUESTION PAPERS | NEWS | RESULTS | FORUMS

f1 (x) = 0

∴f1 (c) = 0
(iii) The Slope of the line AB is equal to Zero, i.e., the line AB is parallel to x – axis.

∴ f (a) = f (b)

3. Verify Rolle’s Theorem for the function f (x) = x2 – 4x + 8 in the internal [1,3]

Solution: We know that every Poly nominal is continuous and differentiable for all points and hence f
(x) is continuous and differentiable in the internal [1,3].

Also f (1) = 1 – 4 + 8 = 5, f (3) = 32 – 43 + 8 = 5

Hence f (1) = f (3)


Thus f (x) satisfies all the conditions of the Rolle’s Theorem. Now f1 (x) = 2x – 4 and f1 (x) = 0
⇒ 2x – 4 = 0 or x = 2. Clearly 1 < 2 < 3. Hence there exists 2t (1,3) such that f1 (2) = 0. This shows
that Rolle’s Theorem holds good for the given function f (x) in the given interval.

−x
4. Verify Rolle’s Theorem for the function f (x) = x (x + 3) e 2
in the interval [-3, 0]

Solution: Differentiating the given function W.r.t ‘x’ we get

⎛ 1 ⎞ −x −x
f 1 ( x) = ( x 2 + 3 x)⎜ − ⎟e 2 + (2 x + 3)e 2
⎝ 2⎠
1 −x
= − ( x 2 − x − 6) e 2
2
∴ f (x) exists (i.e finite) for all x and hence continuous for all x.
1

Also f (-3) = 0, f (0) = 0 so that f (-3) = f (0) so that f (-3) = f (0). Thus f (x) satisfies all the conditions of
the Rolle’s Theorem.

Now, f1 (x) = 0

1 −x
⇒ − ( x 2 − x − 6) e 2 = 0
2
Solving this equation we get x = 3 or x = -2

Clearly –3 < -2 < 0. Hence there exists –2∈ (-3,0) such that f1 (-2) = 0

This proves that Rolle’s Theorem is true for the given function.

www.bookspar.com | VTU NOTES | QUESTION PAPERS 22 of 124


www.bookspar.com | VTU NOTES | QUESTION PAPERS | NEWS | RESULTS | FORUMS

2011

5. Verify the Rolle’s Theorem for the function Sin x in [-π, π]

Solution: Let f (x) = Sin x

Clearly Sinx is continuous for all x.

Also f1 (x) = Cos x exists for all x in (-π, π) and f (-π) = Sin (-π) = 0; f (π) = Sin (π) = 0 so that f (-π) = f
(π)

Thus f (x) satisfies all the conditions of the Rolle’s Theorem .

Now f1 (x) = 0 ⇒ Cos x = 0 so that

π
X=±
2
π
Both these values lie in (-π,π). These exists C = ±
2
Such that ( c ) = 0
f1

Hence Rolle’s theorem is vertified.

6. Discuss the applicability of Rolle’s Theorem for the function f (x) = ІxІ in [-1,1].

Solution: Now f (x) = ⏐x⏐= x for 0 ≤ x ≤ 1


-x for –1 ≤ x ≤ 0

f (x) being a linear function is continuous for all x in [-1, 1]. f(x) is differentiable for all x in (-
1,1) except at x = 0. Therefore Rolle’s Theorem does not hold good for the function f (x) in [-1,1].
Graph of this function is shown in figure. From which we observe that we cannot draw a tangent to the
curve at any point in (-1,1) parallel to the x – axis.

Y
y = ⏐x⏐

x
-1 0 1

www.bookspar.com | VTU NOTES | QUESTION PAPERS 23 of 124


www.bookspar.com | VTU NOTES | QUESTION PAPERS | NEWS | RESULTS | FORUMS

Exercise:
7. Verify Rolle’s Theorem for the following functions in the given intervals.

a) x2 – 6x + 8 in [2,4]

b) (x – a)3 (x – b)3 in [a,b]

⎧ x 2 + ab ⎫
c) log ⎨ ⎬ in [a,b]
⎩ ( a + b) x ⎭
8. Find whether Rolle’s Theorem is applicable to the following functions. Justify your
answer.

a) f (x) = ⏐x – 1 ⏐ in [0,2]

b) f (x) = tan x in [0, π] .

9. State & prove Lagrange’s (1st) Mean Value Theorem with Geometric meaning.

Statement: Let f (x) be a function of x such that

(i) If is continuous in [a,b]

(ii) If is differentiable in (a,b)

Then there exists atleast a point (or value) C∈ (a,b) such that.
f (b ) − f ( a )
f 1 (c ) =
b−a

i.e., f (b) = f (a) + (b – a) f1 (c)


Proof:

[b,f(b)]

[a,f(a)]

x
a c b

www.bookspar.com | VTU NOTES | QUESTION PAPERS 24 of 124


www.bookspar.com | VTU NOTES | QUESTION PAPERS | NEWS | RESULTS | FORUMS

Define a function g (x) so that g (x) = f (x) – Ax ---------- (1)

Where A is a Constant to be determined.

So that g (a) = g (b)

Now, g (a) = f (a) – Aa

G (b) = f (b) – Ab

∴ g (a) = g (b) ⇒ f (a) – Aa = f (b) – Ab.

f (b) − f ( a )
i.e., A = ---------------- (2)
b−a
Now, g (x) is continuous in [a,b] as rhs of (1) is continuous in [a,b]
G(x) is differentiable in (a,b) as r.h.s of (1) is differentiable in (a,b).

Further g (a) = g (b), because of the choice oif A.

Thus g (x) satisfies the conditions of the Rolle’s Theorem.

∴ These exists a value x = c sothat a < c < b at which g1 ( c ) = 0

∴Differentiate (1) W.r.t ‘x’ we get

g1 (x) = f 1 (x) – A

∴ g1 ( c ) = f1 ( c )- A (∵ x =c)

⇒ f 1 ( c ) - A = 0 (∵g1 ( c ) = 0)
∴f1 ( c ) = A -------------- (3)

From (2) and (3) we get

f (b ) − f ( a )
f 1 (c ) = (or) f (b) = f (a) + (b – a) f1 (c) For a < c < b
b−a

Corollary: Put b – a = h
i.e., b = a + h and c = a + θ h
Where 0 < θ < 1
Substituting in f (b) = f (a) + (b – a) f1 ( c )

∴ f (a + h) = f (a) + h f (a + θ h), where 0 < θ < 1.

www.bookspar.com | VTU NOTES | QUESTION PAPERS 25 of 124


www.bookspar.com | VTU NOTES | QUESTION PAPERS | NEWS | RESULTS | FORUMS

Geometrical Interpretation:-
Since y = f (x) is continuous in [a,b], it has a graph as shown in the figure below,
At x = a, y = f (a)
At x = b, y = f (b)

Y B Y B
P
α
A C A
Q

x X
0 a c b 0 Figure (ii)

Figure (i)

Slope of the line joining the points A (a,f(a)) and B ( (b,f (b))

f (b) − f (b)
Is (∴ Slope = m = tan θ)
b−a

= tan α

Where α is the angle mode by the line AB with x – axis

= Slope of the tangent at x = c

= f1 ( c ), where a < c < b

Geometrically, it means that there exists at least are value of x = c, where a < c < b at which the
tangent will be parallel to the line joining the end points at x = a & x = b.

Note: These can be more than are value at which the tangents are parallel to the line joining points A &
B (from Fig (ii)).

www.bookspar.com | VTU NOTES | QUESTION PAPERS 26 of 124


www.bookspar.com | VTU NOTES | QUESTION PAPERS | NEWS | RESULTS | FORUMS

10. Verify Lagrange’s Mean value theorem for f(x) = (x – 1) (x – 2) (x – 3) in [0,4].

Solution: Clearly given function is continuous in [0,4] and differentiable in (0,4), because f (x) is in
polynomial.

f (x) = (x – 1) (x – 2) (x – 3)

f (x) = x3 – 6x2 + 11x – 6

and f (0) = 03 – 6(0)2 + 11 (0) – 6 = -6

f(4) = 43 – 6 (4)2 + 11 (4) – 6 = 6


Differentiate f (x) W.r.t x, we get

F1 (x) = 3x2 – 6x + 11

Let x = c, f1 ( c) = 3c2 – 6c + 11

By Lagrange’s Mean value theorem, we have

f (b) − f (a) f (4) − f (0)


f 1 (c ) = =
b−a (4 − 0)
6 − ( − 6)
= =3
4

∴3c2 – 6c + 11 = 3

⇒ 3c2 – 6c + 8 = 0

Solving this equation, we get

2
C=2± ∈ (0,4)
3
Hence the function is verified.

11. Verify the Lagrange’s Mean value theorem for f (x) = logx in [1,e].

Solution: Now Logx is continuous for all x > 0 and hence [1,e].
1
Also f 1 ( x) = which exists for all x in (1,e)
x
Hence f (x) is differentiable in (1,e)

∴by Lagrange’s Mean Value theorem, we get

www.bookspar.com | VTU NOTES | QUESTION PAPERS 27 of 124


www.bookspar.com | VTU NOTES | QUESTION PAPERS | NEWS | RESULTS | FORUMS

Loge − Log1 1 1 1
= ⇒ =
e −1 c e −1 c
⇒C=e–1

⇒1<e- <2<e

Since e∈ (2,3)
∴ So that c = e – 1 lies between 1 & e

Hence the Theorem.

12. Find θ for f (x) = Lx2 + mx + n by Lagrange’s Mean Value theorem.

Solution: f (x) = Lx2 + mx + n

∴f1 (x) = 2 Lx + m

We have f (a + h) = f (a) + hf1 (a + θ h)

Or f (a + h) – f (a) = hf1 (a + θ h)

i.e., { (a + h)2 + m (a + h) + n} – { a2 + ma + n} = h {2 (a + θ h) + m}

Comparing the Co-efficient of h2, we get


1
1 = 2θ ∴θ = ∈ (0,1)
2
Exercise:
⎡ π⎤
13. Verify the Lagrange’s Mean Value theorem for f (x) = Sin2x in ⎢0, ⎥
⎣ 2⎦
b−a b−a
14. Prove that, < tan-1 b – tan-1 a < if 0 < a < b and reduce that
1+ b 2
1+ a2
π 3 4 π 1
+ < tan −1 < +
4 25 3 4 6
2 Sinx ⎛ π⎞
15. Show that < < 1 in ⎜ 0, ⎟
π x ⎝ 2⎠

b−a b−a
16. Prove that < Sin −1b − Sin −1 a < Where a < b. Hence reduce
1− a2 1− b2
π 1 1 π 1
− < Sin −1 < − .
6 2 3 4 6 15

www.bookspar.com | VTU NOTES | QUESTION PAPERS 28 of 124


www.bookspar.com | VTU NOTES | QUESTION PAPERS | NEWS | RESULTS | FORUMS

2011

17. State & prove Cauchy’s Mean Value Theorem with Geometric meaning.

Proof: The ratio of the increments of two functions called Cauchy’s Theorem.
Statement: Let g (x) and f (x) be two functions of x such that,

(i) Both f (x) and g (x) are continuous in [a,b]

(ii) Both f (x) and g (x) are differentiable in (a,b)

(iii) g1 (x) ≠ 0 for any x ε (a,b)

These three exists at least are value x = c ε (a,b) at which

f 1 (c) f (b) − f (a)


=
g 1 (c) g (b) − g (a )

Proof: Define a function,

φ (x) = f (x) – A. g (x) ------------------ (1)

So that φ (a) = φ (b) and A is a Constant to be determined.

Now, φ (a) = f (a) – A g (a)

φ (b) = f (b) – A. g (b)

∴ f (a) – A g (a) = f (b) – A. g (b)


f (b) − f (b)
⇒ A= -------------------- (2)
g (b) − g (a )

Now, φ is continuous in [a,b] as r.h.s of (1) is continuous in [a,b] and φ (x) is differentiable in
(a,b) as r.h.s of (1) is differentiable in [a,b].

Also φ (a) = φ (b)

Hence all the conditions of Rolle’s Theorem are satisfied then there exists a value x = c ∈ (a,b)
such that φ1 ( c ) = 0.

Now, Differentiating (1) W.r.t x, we get

φ1 (x) = f1 (x) – A.g1 (x)

at x = c ∈ (a,b)

www.bookspar.com | VTU NOTES | QUESTION PAPERS 29 of 124


www.bookspar.com | VTU NOTES | QUESTION PAPERS | NEWS | RESULTS | FORUMS

∴φ1 ( c ) = f1 ( c ) – A g1 (c)

0 = f1 ( c) – A g1 ( c ) (∵ g1 (x) ≠ 0)

f 1 (c )
⇒ A= --------------- (3)
g 1 (c )
Substituting (3) in (2), we get
f 1 (c) f (b) − f (a)
= , where a < c < b
g 1 (c) g (b) − g (a )
Hence the proof.

18. Verify Cauchy’s Mean Value Theorem for the function f (x) = x2 + 3, g (x) = x3 + 1 in [1,3]

Solution: Here f (x) = x2 + 3, g (x) = x3 + 1

Both f (x) and g (x) are Polynomial in x. Hence they are continuous and differentiable for all x
and in particular in [1,3]

Now, f1 (x) = 2x, g1 (x) = 3x2

Also g 1 (x) ≠ 0 for all x ∈ (1,3)

Hence f (x) and g (x) satisfy all the conditions of the cauchy’s mean value theorem.
Therefore

f (3) − f (1) f 1 (c)


= , for some c : 1 < c < 3
g (3) − g (1) g 1 (c)

12 − 4 26
i.e., =
28 − 2 3c 2

2 1 13 1
i.e., = ⇒C = =2
13 3c 6 6

1
Clearly C = 2 lies between 1 and 3.
6

Hence Cauchy’s theorem holds good for the given function.

www.bookspar.com | VTU NOTES | QUESTION PAPERS 30 of 124


www.bookspar.com | VTU NOTES | QUESTION PAPERS | NEWS | RESULTS | FORUMS

2011

⎡ π⎤
19. Verify Cauchy’s Mean Value Theorem for the functions f (x) Sin x, g (x) = Cos x in ⎢0, ⎥
⎣ 2⎦

Solution: Here f (x) = Sin x, g (x) = Cos x so that

f1 (x) = Cos x ,g1 (x) = - Sinx

⎡ π⎤
(
Clearly both f (x) and g (x) are continuous in ⎢0, ⎥ , and differentiable in 0, π
⎣ 2⎦ 2
)
(
Also g1 (x) = -Sin x ≠ 0 for all x ∈ 0, π
2
)
∴ From cauchy’s mean value theorem we obtain

⎛π ⎞
f ⎜ ⎟ − f (0)
⎝2⎠ f 1 (c ) π
= 1 for some C : 0 < C <
⎛π ⎞ g (c ) 2
g ⎜ ⎟ − g (0)
⎝2⎠

1 − 0 Cosc
i.e., = i.e., -1 = - Cot c (or) Cot c = 1
0 − 1 − Sinc

π π π
∴C= , clearly C = lies between 0 and
4 4 2

Thus Cauchy’s Theorem is verified.

Exercises:

20. Find C by Cauchy’s Mean Value Theorem for

a) f (x) = ex, g (x) = e-x in [0,1]

b) f (x) = x2, g (x) = x in [2,3]

21. Verify Cauchy’s Mean Value theorem for

⎡ 1 ⎤
a) f (x) = tan-1 x, g (x) = x in ⎢ ,1⎥
⎣ 3 ⎦
1
b) f(x) = log x, g (x) = in [1,e]
x

31
www.bookspar.com | VTU NOTES | QUESTION PAPERS 31 of 124
www.bookspar.com | VTU NOTES | QUESTION PAPERS | NEWS | RESULTS | FORUMS

2011

Generalized Mean Value Theorem:

22. State Taylor’s Theorem and hence obtain Maclaurin’s expansion (series)

Statement: If f (x) and its first (n – 1) derivatives are continuous in [a,b] and its nth derivative exists in
(a,b) then

(b − a) 2 11 (b − a ) n −1 n-1 (b − a) n n
f(b) = f(a) + (b – a) (a) +
f1 f (a) + ---------+ f (a) + f (c )
2! (n − 1)! n!

Where a < c < b

Remainder in Taylor’s Theorem:

We have
( x − a) 2 11 ( x − a) n −1 n-1
f (x) = f (a) + (x – a) f 1 (a) + f (a) + --------- + f (a) +
2! (n − 1)!
( x − a) n n
f [a + (x – a) θ}
∠n

f (x) = S n (x) + R n (x)

( x − a) n n
Where R n (x) = f [a + (x – a) θ] is called the Largrange’s form of the Remainder.
n!
xn n
Where a = 0, R n (x) = f (θx), 0 < θ < 1
n!
Taylor’s and Maclaurin’s Series:

We have f (x) = Sn (x) + Rn (x)

∴ Lim[ f ( x ) − S n ( x )] = Lim Rn ( x )
n →∞ n→∞

If Lim Rn ( x) = 0thenf ( x) Lim S n ( x )


n →∞ n →∞

Thus Lim S n ( x) converges and its sum is f (x).


n →∞

This implies that f (x) can be expressed as an infinite series.

( x − a) 2 11
i.e., f (x) = f (a) + (x – a) f 1 (a) + f (a) + ---------- to ∞
2!
This is called Taylor’s Series.

www.bookspar.com | VTU NOTES | QUESTION PAPERS 32 of 124


www.bookspar.com | VTU NOTES | QUESTION PAPERS | NEWS | RESULTS | FORUMS

Putting a = 0, in the above series, we get


( x) 2 11
F (x) = f (0) + x f 1 (0) + f (0) + --------- to ∞
2!
This is called Maclaurin’s Series. This can also denoted as
( x) 2 ( x) n
Y = y (0) + x y 1 (0) + y 2 (0) + -------- + y n (0) ---------- to ∞
2! n!
Where y = f (x), y1 = f 1 (x), -------------- y n = f n (x)

23. By using Taylor’s Theorem expand the function e x in ascending powers of (x – 1)

Solution: The Taylor’s Theorem for the function f (x) is ascending powers of (x – a) is
( x − a ) 2 11
f (x) = f (a) + (x – a) f 1 (a) + f (a) + ------------ (1)
∠2
Here f (x) = e x and a = 1
f 1 (x) = e x ⇒ f 1 (a) = e
f 11 (x) = e x ⇒ f 11 (a) = e

∴ (1) becomes
( x − 1) 2
e = e + (x – 1) e +
x e + -------------
2
( x − 1) 2
= e { 1 + (x – 1) + + --------}
2

24. By using Taylor’s Theorem expand log sinx in ascending powers of (x – 3)

Solution: f (x) = Log Sin x, a = 3 and f (3) = log sin3

Cosx
Now f 1 (x) = = Cotx, f 1 (3) = Cot3
Sinx

f 11 (x) = - Cosec 2x, f 11 (3) = - Cosec 23


f 111 (x) = - 2Cosecx (-Cosecx Cotx) = 2Cosec 3x Cotx

∴ f 111 (3) = 2Cosec 33 Cot3


( x − a) 2 11 ( x − a)3 111
∴ f (x) = f (a) + (x – a) f 1 (a) + f (a) + f (a) + ------------
2! 3!

( x − 3) 2 11 ( x − 3)3 111
∴Log Sinx = f (3) + (x – 3) f 1(3) + f (3) + f (3) + -----------
2! 3!

( x − 3) 2 ( x − 3)3
= logsin3 + (x – 3) Cot3 + (-Cosec 3) +
2 2 Cosec 33Cot3 + ---
2! 3!

www.bookspar.com | VTU NOTES | QUESTION PAPERS 33 of 124


www.bookspar.com | VTU NOTES | QUESTION PAPERS | NEWS | RESULTS | FORUMS

2011

Exercise:
⎛ π⎞
25. Expand Sinx is ascending powers of ⎜ x − ⎟
⎝ 2⎠
26. Express tan x in powers of (x – 1) up to the term containing (x – 1) 3
–1

27. Apply Taylor’s Theorem to prove

⎡ h 2 h3 ⎤
e x + h = e x ⎢1 + h + + + − − − − − − − − −⎥
⎣ 2! 3! ⎦

Problems on Maclaurin’s Expansion:

28. Expand the log (1 + x) as a power series by using Maclaurin’s theorem.

Solution: Here f (x) = log (1 + x), Hence f (0) = log 1 = 0

We know that

dn d n −1 ⎧ 1 ⎫
f n ( x) = {log(1 + x )} = ⎨ ⎬
dx n dx n −1 ⎩1 + x ⎭

(−1) n −1 (n − 1)!
= , n = 1,2,---------
(1 + n) n

Hence f n (0) = (-1) n-1 (n – 1) !

f 1 (0) = 1, f 11 (0) = -1, f 111 (0) = 3!, f 1v (0) = - 3!

Substituting these values in

x 2 11 xn n
f (x) = f (0) + x f (0) +
1 f (0) + -------- + f (0) + ------------
2! n!

x2 x3 x4
∴ log (1+x) = 0 + x . 1 + (-1) + 2! + - 3! + ---------------
2! 3! 4!

x2 x3 x4
=x- + - + ---------
2 3 4
This series is called Logarithmic Series.

www.bookspar.com | VTU NOTES | QUESTION PAPERS 34 of 124


www.bookspar.com | VTU NOTES | QUESTION PAPERS | NEWS | RESULTS | FORUMS

2011

29. Expand tan –1 x by using Macluarin’s Theorem up to the term containing x 5


Solution: let y = tan –1 x, Hence y (0) = 0
1
We find that y 1 = which gives y 1 (0) = 1
1+ x2
Further y 1 (1 + x 2) = 1, Differentiating we get

Y 1 . 2x + (1 + x 2) y 2 = 0 (or) (1 + x2) y 2 + 2xy 1 = 0


Hence y 2 (0) = 0
Taking n th derivative an both sides by using Leibniz’s Theorem, we get
n( n − 1)
(1 + x 2) y n + 2 + n . 2xy n +1 + . 2. y n + 2xy n – 1 + n.2.y n = 0
1 .2
i.e., (1 + x 2) y n + 2 + 2 (n +1) x y n + 1 + n (n + 1) y n = 0

Substituting x = 0, we get, y n + 2 (0) = -n (n + 1) y n (0)

For n = 1, we get y 3 (0) = - 2y 1 (0) = - 2

For n = 2, we get y 4 (0) = - 2 .3.y 2 (0) = 0

For n = 3, we get y 5 (0) = - 3.4.y 3 (0) = 24


Using the formula
x2 x3
Y = y (0) + x y 1 (0) + y 2 (0) + y 3 (0) + ---------
2! 3!
x3 x5
We get tan –1 x = x - + - -------------
3 5
Exercise:

30. Using Maclaurin’s Theorem prove the following:


x2 5x 4
a) Secx = 1 + + + --------
2! 4!
x 3 3x 5
b) Sin –1 x = x + + + --------------
6 40
x3
c) e x Cos x = 1 + x - + ---------------
3

d) Expand e ax Cos bx by Maclaurin’s Theorem as far as the term containing x 3

www.bookspar.com | VTU NOTES | QUESTION PAPERS 35 of 124


www.bookspar.com | VTU NOTES | QUESTION PAPERS | NEWS | RESULTS | FORUMS

2011

Exercise : Verify Rolle’s Theorem for

⎡ π 5π ⎤
(i) f ( x) = e x (sin x − cos x) in ⎢ , ⎥ ,
⎣4 4 ⎦

(ii) f ( x) = x( x − 2)e x / 2 in [0,2]

sin 2 x ⎡ π 5π ⎤
(iii) f ( x) = in ⎢ , ⎥ .
e2 x ⎣4 4 ⎦

Exercise : Verify the Lagrange’s Mean Value Theorem for


⎡ 1⎤
(i) f ( x ) = x ( x − 1)( x − 2) in ⎢ 0, ⎥
⎣ 2⎦

(ii) f ( x) = Tan −1x in [ 0,1]

Exercise : Verify the Cauchy’s Mean Value Theorem for

1 ⎡1 ⎤
(i) f ( x ) = x and g ( x) = in ⎢ ,1⎥
x ⎣4 ⎦

1
in [ a, b]
1
(ii) f ( x) = and g ( x) =
2 x
x

(iii) f ( x ) = Sin x and g ( x ) = Cos x in [ a, b]

www.bookspar.com | VTU NOTES | QUESTION PAPERS 36 of 124


www.bookspar.com | VTU NOTES | QUESTION PAPERS | NEWS | RESULTS | FORUMS

2011

UNIT – II

DIFFERENTIAL CALCULUS-II

Give different types of Indeterminate Forms.

If f (x) and g (x) be two functions such that Lim f (x ) and Lim g (x ) both exists, then
x→ a x→ a

f ( x) Lim f ( x)
Lim = x→a
x→a g ( x) Lim g ( x)
x→a

If Lim f (x ) = 0 and Lim g (x ) = 0 then


x→ a x→ a

f ( x) 0
Lim = Which do not have any definite value, such an expression is called
x →a g ( x) 0

indeterminate form. The other indeterminate forms are ,0 × ∞, ∞ − ∞ , 00, ∞0 and 1 ∞

1. State & prove L’ Hospital’s Theorem (rule) for Indeterminate Forms.

0 ∞
L’Hospital rule is applicable when the given expression is of the form or
0 ∞

Statement: Let f (x) and g (x) be two functions such that

(1) Lim f (x ) = 0 and Lim g (x ) = 0


x→ a x→ a

(2) f1 (a) and g1 (a) exist and g1 (a) ≠ 0

f ( x) Lim f 1 ( x)
Then Lim = x →a
x→a g ( x) Lim g 1 ( x)
x→a

⎡ 1 x ⎤
= Lim ⎢
f ( x) g ⎥ 0
Proof: Now Lim , which takes the indeterminate form . Hence applying the
x →a g ( x) x→a ⎢ 1 ⎥ 0
⎢⎣ f ( x) ⎥⎦
L’ Hospitals theorem, we get

www.bookspar.com | VTU NOTES | QUESTION PAPERS 37 of 124


www.bookspar.com | VTU NOTES | QUESTION PAPERS | NEWS | RESULTS | FORUMS

2011
− g 1 ( x) 2
f ( x) [ g ( x)]2 g 1 ( x) ⎡ f ( x) ⎤
Lim = Lim = Lim ⎢ ⎥
x →a − f 1 ( x) x →a f 1 ( x) g ( x)
x →a g ( x)
2
⎣ ⎦
[ f ( x)]

2
⎡ g 1 ( x) ⎤ ⎡ f ( x) ⎤
= ⎢ Lim 1 ⎥ ⎢ Lim ⎥
⎣ x →a f ( x) ⎦ ⎣ x→ a g ( x) ⎦

f ( x)
If Lim ≠ 0and ≠ ∞ then
x→a g ( x)

⎡ g 1 ( x) ⎤ ⎡ f ( x) ⎤
1 = ⎢ Lim 1 ⎥ ⎢ Lim ⎥
⎣ x→a f ( x) ⎦ ⎣ x→a g ( x) ⎦

f ( x) f 1 ( x)
i.e Lim = Lim 1
x→a g ( x) x →a g ( x)

f ( x)
If Lim = 0 or ∞ the above theorem still holds good.
x→ a g ( x)

Sinx 0
2. Evaluate Lim = form
x→a x 0

Solution: Apply L’Hospital rule, we get

Cosx Cosθ 1
Lim = = =1
x→a 1 1 1

Sinx
∴ Lim =1
x →a x

log Sinx
3. Evaluate Lim
x→a Cotx
log Sinx log Sin0 log 0 − ∞
Solution: Lim = = = form
x→a Cotx Cot 0 ∞ ∞

Apply L’ Hospital rule

− Cosce 2 x
= Lim
x→a − 2Co sec xCo sec xCotx

www.bookspar.com | VTU NOTES | QUESTION PAPERS 38 of 124


www.bookspar.com | VTU NOTES | QUESTION PAPERS | NEWS | RESULTS | FORUMS

1
= Lim =0
x→a 2Cotx

log Sinx
∴ Lim =0
x→a Cotx

Exercise: 1

Evaluate

tan x
a) Lim
x →0 x

1
b) Lim(1 + x) x
x →0

a x −1
c) Lim
x →∞ x

xn − an
d ) Lim
x →0 x−a

4. Explain ∞ - ∞ and 0 × ∞ Forms:

Solution: Suppose Lim f (x) = 0 and Lim g (x) = ∞ in this case


x→ a x→a

0 ∞
Lim f (x) - g(x) = 0 × ∞, reduce this to or form
x→ a 0 ∞
⎧ ⎫
⎪ f ( x) ⎪ 0
Let Lim[ f ( x).g ( x)] = Lim ⎨ ⎬ = form
⎪ 1 g ( x) ⎪ 0
x→a x→a
⎩ ⎭
⎧ ⎫
⎪ g ( x) ⎪ ∞
Or Lim[ f ( x).g ( x)] = Lim ⎨ ⎬= form
x→a x→a 1
⎪ f ( x) ⎪ ∞
⎩ ⎭
L’ Hospitals rule can be applied in either case to get the limit.

Suppose Lim f (x) = ∞ and Lim g (x) = ∞ in this case Lim[ f ( x).g ( x)] = ∞ − ∞ form, reduce
x→ a x→a x→a

0 ∞
this or form and then apply L’Hospitals rule to get the limit
0 ∞

www.bookspar.com | VTU NOTES | QUESTION PAPERS 39 of 124


www.bookspar.com | VTU NOTES | QUESTION PAPERS | NEWS | RESULTS | FORUMS

2011
⎧ 1 log(1 + x) ⎫
5. Evaluate Lim ⎨ − ⎬

x →0 x x2 ⎭

⎧ 1 log(1 + x) ⎫
Solution: Given Lim ⎨ − ⎬ = ∞ - ∞ form

x →0 x x2 ⎭

⎧ x − log(1 + x ⎫ 0
∴ Required limit = Lim ⎨ ⎬ = form
x →0
⎩ x2 ⎭ 0

Apply L’Hospital rule.

1
1−
= Lim 1+ x
x →0 2x

x
= Lim 1 + x = Lim
1 1
=
x →0 2x x → 0 2(1 + x) 2

⎧1 ⎫
6. Evaluate Lim ⎨ − Cotx ⎬

x →0 x

Solution: Given limit is ∞ - ∞ form at x = 0. Hence we have

⎧ 1 Cosx ⎫
Required limit = Lim ⎨ − ⎬

x→0 x Sinx ⎭

⎛ Sinx − xCosx ⎞ ⎛ 0 ⎞
= Lim⎜ ⎟ = ⎜ ⎟ form
x →0
⎝ xSinx ⎠ ⎝0⎠

Apply L’ Hospital’s rule

Cosx − Cosx + xSinx


= Lim
x →0 xCosx + Sinx

xSinx ⎛0⎞
= Lim = ⎜ ⎟ form
x →0 xCosx + Sinx ⎝ 0 ⎠

Apply L’ Hospitals rule

www.bookspar.com | VTU NOTES | QUESTION PAPERS 40 of 124


www.bookspar.com | VTU NOTES | QUESTION PAPERS | NEWS | RESULTS | FORUMS

2011
xCosx + Sinx
= Lim
x →0 Cosx − xSinx + Cosx

0 0
= = =0
2−0 2

7. Evaluate Lim tan x log x


x →0

Solution: Given limit is (0 × - ∞) form at x = 0

log x ⎡ − ∞ ⎤
∴ Required limit = Lim = form
x →0 Cotx ⎢⎣ ∞ ⎥⎦

Apply L’ Hospitals rule

1
= Lim x
x →0 − Co sec 2 x

− Sin 2 x ⎛ 0 ⎞
= Lim ⎜ ⎟ form
x →0 x ⎝0⎠

Apply L Hospitals rule

− 2 SinxCosx
= Lim =0
x→0 1

⎛ πx ⎞
8. Evaluate Lim Sec⎜ ⎟. log x
x →1
⎝ 2⎠
Solution: Given limit is (∞ × 0) form at x = 1

log x 0
∴ Required limit = Lim form
x →1 πx 0
Cos
2

Apply L’ Hospitals rule

1
x 2
= Lim =−
x →1 πx π π
− Sin .
2 2

www.bookspar.com | VTU NOTES | QUESTION PAPERS 41 of 124


www.bookspar.com | VTU NOTES | QUESTION PAPERS | NEWS | RESULTS | FORUMS

2011

Exercise: 2

Evaluate

⎛ x 1 ⎞
a) Lim⎜⎜ − ⎟
x →1 x − 1
⎝ log x ⎟⎠

⎛a ⎛ x ⎞⎞
b) Lim⎜⎜ − Cot ⎜ ⎟ ⎟⎟

x →0 x
⎝ a ⎠⎠

⎛ 1 ⎞
c) Lim⎜ Secx − ⎟
x→ ⎝
π 1 − Sinx ⎠
2

d) Lim⎛⎜ a x − 1⎞⎟ x
1

x →∞ ⎝ ⎠
9. Explain Indeterminate Forms 0 0 , 1∞ , ∞ 0 , 0 ∞

Solution: At x = a, [ f ( x)] takes the indeterminate form


g ( x)

(i) 0 0 if Lim f(x) = 0 and Lim g (x) = 0


x→a x→a

(ii) 1∞ if Lim f(x) = 1 and Lim g (x) = ∞


x→a x→a

(iii) ∞ 0 if Lim f(x) = ∞ and Lim g(x) = 0 and


x→a x→a

(iv) 0 ∞ if Lim f(x) = 0 and Lim f (x) = ∞


x→a x→a

In all these cases the following method is adopted to evaluate Lim [ f ( x)]
g ( x)
x→a

Let L = Lim [ f ( x)] so that


g ( x)
x→a

Log L = Lim [g(x) log f (x)] = 0 × ∞


x→a

0 ∞
Reducing this to or and applying L’ Hospitals rule, we get Log L = a Or
0 ∞
L = ea

www.bookspar.com | VTU NOTES | QUESTION PAPERS 42 of 124


www.bookspar.com | VTU NOTES | QUESTION PAPERS | NEWS | RESULTS | FORUMS

2011

10. Evaluate Lim x Sinx


x→0

Solution: let L = Lim x Sinx ⇒ 00 form at x = 0


x→0

Hence Log L = Lim Sinx log x ⇒ 0 × ∞ form


x→0

log x log x ⎛ ∞ ⎞
∴ LogL = Lim = Lim ⎜ ⎟ form
x → 0 Coscex ∞
x→0 1 ⎝ ⎠
Sinx

Apply L’ Hospital rule,

( )
1
x − Sinx. tan x 0
= Lim = Lim form
x →0 − CoscexCotx x→0 x 0

Apply L’ Hospitals rule we get

sin xSec 2 x − Cosx tan x


= Lim = −∞
x →0 1

1 1
∴ LogL = −∞ ⇒ L = e −∞ = ∞
= =0
e ∞

∴L=0
1
11. Evaluate Lim( x) 1− x
x →1

1 −x
Solution: let L = Lim( x) 1
is 1∞ form
x →1

⎛ 1 ⎞ ⎛0⎞
∴ LogL = Lim⎜ log x ⎟ ≡ ⎜ ⎟ form
x →1 1 − x
⎝ ⎠ ⎝0⎠

Apply L’ Hospitals rule

1
1
= Lim x = Lim = −1
x →1 − 1 x →1 − x

www.bookspar.com | VTU NOTES | QUESTION PAPERS 43 of 124


www.bookspar.com | VTU NOTES | QUESTION PAPERS | NEWS | RESULTS | FORUMS

2011
∴ Log L = -1

1
⇒ L = e −1 =
e
1
⎛ tan x ⎞ x2
12. Evaluate Lim⎜ ⎟
x →0
⎝ x ⎠
1
⎛ tan x ⎞ x2
Solution: let L = Lim⎜ ⎟ ≡ 1∞ form
x →0
⎝ x ⎠

⎧1 ⎛ tan x ⎞⎫
∴ LogL = Lim ⎨ 2 log⎜ ⎟⎬ ≡ (∞ × 0) form

x →0 x
⎝ x ⎠⎭

∴ LogL = Lim ⎨
⎧ log tan x
⎪ (x
)⎫⎪ ≡ ⎛ 0 ⎞ form
⎬ ⎜ ⎟
⎝0⎠
x →0 2
⎪⎩ x ⎪⎭

Apply L’ Hospitals rule

⎧ xSec 2 x − tan x ⎫
⎪⎪ 1 x2 ⎪⎪
= Lim ⎨ ⎬
2x
⎪ tan x
x→0

⎩⎪ x ⎭⎪

1 ⎡ xSec 2 x − tan x ⎤ ⎛ 0 ⎞
LogL = Lim ⎢ ⎥ = ⎜ ⎟ form
2 x →0 ⎣ x3 ⎦ ⎝0⎠

Apply L’ Hospital rule, we get

1 Sec 2 x + 2 xSec 2 x tan x − Sec 2 x


= Lim
2 x →0 3x 2

1 ⎛ tan x ⎞
= Lim( Sec 2 x)⎜ ⎟
3 x → 0
⎝ x ⎠

1
Log L =
3
1
∴L = e 3

www.bookspar.com | VTU NOTES | QUESTION PAPERS 44 of 124


www.bookspar.com | VTU NOTES | QUESTION PAPERS | NEWS | RESULTS | FORUMS

2011

Exercise: 3
Evaluate the following limits.

⎛ πx ⎞
tan ⎜ ⎟
⎛ x⎞ ⎝ 2a ⎠
a) Lim( Secx) Cotx b) Lim⎜ 2 − ⎟
x →0 x→a
⎝ a⎠

1
(1 + x) x − e ∞
c) Lim d) Lim(Cosax) x2
x→0 x x →0

e x − e− x − 2 log(1 + x) log(1 + x3 )
(i ) lim (ii ) lim
x →0 x sin x x →0 sin 3 x

1 + sin x − cos x + log(1 − x ) log sin x


(iii ) lim (iv) lim
x →0 x tan 2 x π π
x→
2 ( x − )2
2

cosh x + log(1 − x) − 1 + x sin x sin −1 x


(v) lim (vi ) lim
x →0 x2 π x2
x→
2

e2 x − (1 + x) 2
(vii ) lim
x → 0 x log(1 + x )

Evaluate the following limits.

⎛ x⎞
(i ) lim ⎜ 2 − ⎟ cot( x − a ) (ii ) lim ( cos ecx − cot x )
x→a
⎝ a⎠ x →0

⎡ π ⎤ ⎛ 1 ⎞
(iii ) lim ⎢ x tan x − sec x ⎥ (iv) lim ⎜ cot 2 x − 2 ⎟
π
x→ ⎣ 2 ⎦ x →0
⎝ x ⎠
2

⎡1 1 ⎤
(v) lim ⎢ 2 − (vi ) lim [ 2 x tan x − π sec x ]

x →0 x x tan x ⎥⎦ x→
π
2
1
⎛ 1 + cos x ⎞ x2
b
2
(i ) lim(cos ax) x (ii ) lim ⎜ ⎟
x →0 x →0
⎝ 2 ⎠

www.bookspar.com | VTU NOTES | QUESTION PAPERS 45 of 124


www.bookspar.com | VTU NOTES | QUESTION PAPERS | NEWS | RESULTS | FORUMS

2011
1
⎛ sin x ⎞
1
x2
2 log(1− x )
(iii ) lim(1 − x ) (iv) lim ⎜ ⎟
x →1 x →0
⎝ x ⎠

(iv ) lim (1 + sin x )


cot x
(v ) lim(sin x ) tan x
x →0 x →0

(viii ) lim ( tan x )


2 tan 2 x
(vii ) lim(cos x)cos ec x
x →0 π
x→
4
1
ax + 1 x ⎛ ax + bx + cx ⎞ x
(ix) lim( ) ( x) lim ⎜ ⎟
x →∞ ax − 1 x →0
⎝ 3 ⎠

Evaluate the following limits.


1
⎛ 1 + cos x ⎞ x2
b
2
(i ) lim(cos ax) x (ii ) lim ⎜ ⎟
x →0 x →0
⎝ 2 ⎠

1
⎛ sin x ⎞
1
x2
2 log(1− x )
(iii ) lim(1 − x ) (iv) lim ⎜ ⎟
x →1 x →0
⎝ x ⎠

(iv ) lim (1 + sin x )


cot x
(v ) lim(sin x ) tan x
x →0 x →0

(viii ) lim ( tan x )


2 tan 2 x
(vii ) lim(cos x)cos ec x
x →0 π
x→
4
1
ax + 1 x ⎛ ax + bx + cx ⎞ x
(ix) lim( ) ( x) lim ⎜ ⎟
x →∞ ax − 1 x →0
⎝ 3 ⎠

www.bookspar.com | VTU NOTES | QUESTION PAPERS 46 of 124


www.bookspar.com | VTU NOTES | QUESTION PAPERS | NEWS | RESULTS | FORUMS

2011

Polar Curves
If we traverse in a hill section where the road is not straight, we often see caution boards hairpin
bend ahead, sharp bend ahead etc. This gives an indication of the difference in the amount of bending
of a road at various points which is the curvature at various points. In this chapter we discuss about the
curvature, radius of curvature etc.

Consider a point P in the xy-Plane.


r = length of OP= radial distance
θ = Polar angle
( r, θ)→ Polar co-ordinates
Let r = f (θ) be the polar curve

r = x2 + y2 , θ = tan−1 y ( x) −−−−−−(1)

x = r Cos θ y = r Sin θ
Relation (1) enables us to find the polar co-ordinates
( r, θ) when the Cartesian co-ordinates ( x, y) are known.

Expression for arc length in Cartesian form.

Proof: Let P (x,y) and Q (x + δx, Y + δy) be two neighboring points on the graph of the
function y = f (x). So that they are at length S and S + δs measured from a fixed
point A on the curve.

Y = f (x)

Q
T (Tangent)
δs δy
A δx

N O δx X


From figure, PQ = δS ,

AP = S
∠TPR = ψ and PR =δx, RQ = δy

www.bookspar.com | VTU NOTES | QUESTION PAPERS 47 of 124


www.bookspar.com | VTU NOTES | QUESTION PAPERS | NEWS | RESULTS | FORUMS

2011

∴ Arc PQ = δS

From ∆le PQR, we have

[Chord PQ]2 = PR2 + QR2

[Chord PQ]2 = (δx)2 + (δy)2 (∵ from figure)

When Q is very close to point P, the length of arc PQ is equal to the length of Chord PQ.

i.e arc PQ = Chord PQ = δs

∴ (δs)2 = (δx)2 + (δy)2 -------- (1)

÷ (δx)2, we get

⎛ δs ⎞ ⎛ δy ⎞
2 2

⎜ ⎟ = 1+ ⎜ ⎟
⎝ δx ⎠ ⎝ δx ⎠

When Q → P along the curve, δx → 0, δs → 0

⎛ δs ⎞ ⎛ δy ⎞
2 2

∴ Lim⎜ ⎟ = 1 + Lim⎜ ⎟
δx → 0 δx δx → 0 δ x
⎝ ⎠ ⎝ ⎠

2 2
⎛ ds ⎞ ⎛ dy ⎞
i.e., ⎜ ⎟ = 1 + ⎜ ⎟
⎝ dx ⎠ ⎝ dx ⎠

2
ds ⎛ dy ⎞
∴ = 1+ ⎜ ⎟ --------------(2)
dx ⎝ dx ⎠

Similarly, dividing (1) by δy and taking the limit as δy → 0, we get

2
ds ⎛ dx ⎞
= 1 + ⎜⎜ ⎟⎟ ------------(3)
dy ⎝ dy ⎠

www.bookspar.com | VTU NOTES | QUESTION PAPERS 48 of 124


www.bookspar.com | VTU NOTES | QUESTION PAPERS | NEWS | RESULTS | FORUMS

2011

ds ds
Expressions for &
dx dy
Trace a tangent to the curve at the point P, it makes an angle ψ with the x – axis. From ∆le PRT, we
have

dy
tan ψ =
dx
Q
Equation (2) becomes,
Y

Tangent
T T
A P ψ R ψ
P R

ψ
O M N X

ds
= 1 + tan 2 ψ
dx

= Sec 2ψ = secψ

ds dx
∴ = Sec ψ ( Or) = Cos ψ
dx ds

and equation (3) becomes

ds
= 1 + Cot 2ψ = Co sec 2 ψ = Cosec ψ
dy

ds dy
∴ = Cosecψ or = Sin ψ
dy ds

Derive an expression for arc length in parametric form.

Solution: Let the equation of the curve in Parametric from be x = f (t) and y = g (t).

We have,

www.bookspar.com | VTU NOTES | QUESTION PAPERS 49 of 124


www.bookspar.com | VTU NOTES | QUESTION PAPERS | NEWS | RESULTS | FORUMS

2011

(δs)2 = (δx)2 + (δy)2

÷ by (δt)2, we get

⎛ δs ⎞ ⎛ δx ⎞ ⎛ δy ⎞
2 2 2

⎜ ⎟ =⎜ ⎟ +⎜ ⎟
⎝ δt ⎠ ⎝ δt ⎠ ⎝ δt ⎠

Taking the limit as δt → 0 on both sides, we get

⎛ δs ⎞ ⎛ δx ⎞ ⎛ δy ⎞
2 2 2

Lim⎜ ⎟ = Lim⎜ ⎟ + Lim⎜ ⎟


δt → 0 ⎝ δt ⎠ δt → 0 ⎝ δt ⎠ δt → 0 ⎝ δ t ⎠

2 2 2
⎛ ds ⎞ ⎛ dx ⎞ ⎛ dy ⎞
∴⎜ ⎟ = ⎜ ⎟ + ⎜ ⎟
⎝ dt ⎠ ⎝ dt ⎠ ⎝ dt ⎠

2 2
ds ⎛ dx ⎞ ⎛ dy ⎞
(Or) = ⎜ ⎟ +⎜ ⎟ ------------ (4)
dt ⎝ dt ⎠ ⎝ dt ⎠

Derive an expression for arc length in Cartesian form.

Solution: Let P (r,θ) and Q (r + δr, θ + δθ) be two neighboring points on the graph of the function
r = f (θ). So that they are at lengths S and s + δs from a fixed point A on the curve.

∴ PQ = (S + δs) – s = δs

Draw PN ⊥ OQ

From ∆le OPN,

Q
Tangent
N

r + δr P
φ
r
δθ
θ
X
O

www.bookspar.com | VTU NOTES | QUESTION PAPERS 50 of 124


www.bookspar.com | VTU NOTES | QUESTION PAPERS | NEWS | RESULTS | FORUMS

2011

PN
= Sinδθ
OP

PN
i.e., = Sin δθ (or) PN = r sinδθ
r

ON
and = Cos δθ
OP

ON
i.e = Cos δθ (or) ON = r Cos δθ
r

When Q is very close to P, the length of arc PQ as equal to δs, where δs as the length of chord
PQ.

In ∆le PQN,

(PQ)2 = (PN)2 + (QN)2

but PN = r sin δθ (∵Sin δθ ≈ δθ)

And QN = OQ – ON

= (r + δr) – r Cos δθ

= r + δr – r ( ∵Cos δθ ≈ 1)

∴ QN = δr

And (PQ)2 = (PN)2 + (QN)201

(δS)2 + (rδθ)2 + (δr)2 ---------------- (5)

÷ by (δθ)2 we get

⎛ δS ⎞ ⎛ δr ⎞
2 2

⎜ ⎟ = r +⎜ ⎟
2

⎝ δθ ⎠ ⎝ δθ ⎠
When Q → P along the curve δθ → 0 as δS → 0 and δr → 0

www.bookspar.com | VTU NOTES | QUESTION PAPERS 51 of 124


www.bookspar.com | VTU NOTES | QUESTION PAPERS | NEWS | RESULTS | FORUMS

⎛ δS ⎞ ⎛ δr ⎞
2 2

Lim⎜ ⎟ = r 2 + Lim⎜ ⎟
δθ → 0 ⎝ δθ ⎠ δθ → 0 ⎝ δθ ⎠

2 2
⎛ ds ⎞ ⎛ dr ⎞
i.e ⎜ ⎟ = r +⎜ ⎟
2

⎝ dθ ⎠ ⎝ dθ ⎠

2
ds ⎛ dr ⎞
i.e., = r2 + ⎜ ⎟ --------------- (6)
dθ ⎝ dθ ⎠

Similarly equation (5) ÷ by (δr)2 and taking the limits as δr → 0 we get

⎛ δS ⎞ ⎛ δθ ⎞
2 2

Lim⎜ ⎟ = Lim r 2 ⎜ ⎟ + 1
δr → 0 δ r
⎝ ⎠ δr → 0 ⎝ δr ⎠

⎛ dθ ⎞
2 2
⎛ ds ⎞
i.e ⎜ ⎟ = r 2 ⎜ ⎟ +1
⎝ dr ⎠ ⎝ dr ⎠

⎛ dθ ⎞
2
ds
∴ = r ⎜ 2
⎟ +1 --------------- (7)
dr ⎝ dr ⎠

Note: Angle between Tangent and Radius Vector:-

We have,


Tan φ = r
dr

Sinφ dθ
i.e., =r
Cosφ dr

dθ ds
=r .
ds dr

Sinφ r dθ
= ds
Cosφ dr
ds

dθ dr
∴Sinφ = r and Cos φ =
ds ds

www.bookspar.com | VTU NOTES | QUESTION PAPERS 52 of 124


www.bookspar.com | VTU NOTES | QUESTION PAPERS | NEWS | RESULTS | FORUMS

ds ds
Find and for the following curves:-
dx dy
⎛ x⎞
1) y = C Cos h ⎜ ⎟
⎝c⎠

⎛x⎞
Solution: y = C Cos h ⎜ ⎟
⎝c⎠
Differentiating y w.r.t x. we get

dy ⎛x⎞
= Sin h ⎜ ⎟
dx ⎝c⎠

2
ds ⎛ dy ⎞
∴ = 1+ ⎜ ⎟
dx ⎝ dx ⎠

≡ 1 + sinh 2 x ( c) = Cosh 2 x ( c)
ds ⎛ x⎞
= Cosh ⎜ ⎟
dx ⎝c⎠

Again Differentiating y w.r.t y we get

⎛ x ⎞ 1 dx
1 = C Sin h ⎜ ⎟
⎝ c ⎠ C dy

dx ⎛x⎞
i.e = Cosech ⎜ ⎟
dy ⎝c⎠


ds
dy
= 1 + Co sec h 2 x( )
c

2) x3 = ay2

Solution : x3 = ay2

Differentiating w.r.t y and x separately we get

www.bookspar.com | VTU NOTES | QUESTION PAPERS 53 of 124


www.bookspar.com | VTU NOTES | QUESTION PAPERS | NEWS | RESULTS | FORUMS

2011
dx dy
3x2 = 2 ay and 3 x2 = 2 ay
dy dx

dx 2ay dy 3 x 2
i.e., = and =
dy 3 x 2 dx 2ay

We know that

2
ds ⎛ dy ⎞
= 1+ ⎜ ⎟
dx ⎝ dx ⎠

2
⎛ 3x 2 ⎞
= 1 ⎜⎜
+ ⎟⎟
⎝ 2 ay ⎠

9x3 x 9ay 2 x
= 1+ = 1 +
4a 2 y 2 4a 2 y 2

ds 9x
= 1+ and
dx 4a
2
⎛ dx ⎞
2
ds ⎛ 2ay ⎞
= 1 + ⎜⎜ ⎟⎟ = 1 + ⎜ 2 ⎟
dy ⎝ dy ⎠ ⎝ 3x ⎠

1 1
ds ⎛ 4a 2 y 2 ⎞ 2 ⎛ 4a ⎞ 2
= ⎜1 + ⎟⎟ = ⎜1 + ⎟ (∴x3 = ay2)
dy ⎜⎝ 9x 4 ⎠ ⎝ 9x ⎠

3. y = log cos x
Solution . y= log cos x

Differentiating w.r.t x and y separately, we get

dy 1
= ( – Sin x) = - tan x
dx Cosx

dy
i.e., = - tan x
dx

1 dx
and 1 = (- Sin x)
Cosx dy

www.bookspar.com | VTU NOTES | QUESTION PAPERS 54 of 124


www.bookspar.com | VTU NOTES | QUESTION PAPERS | NEWS | RESULTS | FORUMS

2011

dx dx
i.e., 1 = - tan x (Or) = - Cot x
dy dy

We have

2
⎛ dx ⎞
2
ds ⎛ dy ⎞ ds
= 1+ ⎜ ⎟ and = 1 + ⎜⎜ ⎟⎟
dx ⎝ dx ⎠ dy ⎝ dy ⎠

ds ds
∴ = 1 + tan 2 x & = 1 + Cot 2 x
dx dy

= Sec 2 x
1
ds ds
∴ = Sec x and = (1 + Cot 2 x) 2
dx dy

ds
Find for the following Curves:-
dt
1. x = a (Cos t + t Sin t), y = a (Sin t = t Cos t)

2. x = a Sec t , y = b tan t

(
3. x = a Cost + log tan t
2
), y = a Sin t
Solution of 1
Given x = a (Cos t + t Sin t), y = a (Sin t – t Cos t )

Differentiating x & y W.r.t ‘t’, we get

dx
= a (-Sin t + Sin t + t Cos t)
dt

dx
= a t cos t
dt
dy
and = a (Cos t – Cos t + t Sin t)
dt
dy
= at Sin t
dt
2 2
ds ⎛ dx ⎞ ⎛ dy ⎞
∴ = ⎜ ⎟ +⎜ ⎟
dt ⎝ dt ⎠ ⎝ dt ⎠

www.bookspar.com | VTU NOTES | QUESTION PAPERS 55 of 124


www.bookspar.com | VTU NOTES | QUESTION PAPERS | NEWS | RESULTS | FORUMS

2011

= a 2 t 2 Cos 2 t + a 2 t 2 Sin 2 t
ds
= at
dt
Solution of 2
x = a Sec t, x = b tan t
dx dy
∴ = a Sec t tan t, = b Sec2t
dt dt
We have
2 2
ds ⎛ dx ⎞ ⎛ dy ⎞
∴ = ⎜ ⎟ +⎜ ⎟
dt ⎝ dt ⎠ ⎝ dt ⎠
1
= (a2Sec2 t tan2t + b2 Sec4t) 2
1
= [a2 Sec2 t (Sec2t – 1) + b2 Sec4 t] 2

1
= [a2 Sec4 t– a2 Sec2 t+ b2 Sec4 t] 2

1
ds
= [(a2 + b2) Sec4 t – a2 Sec2t] 2
dt
Solution of 3

x = a (Cos t + log tan t ), y = a Sin t


2
Differentiating x and y w.r.t ‘t’ we get
⎛ ⎞
dx
=a ⎜ − S int + 1 Sec 2 t . 1 ⎟ , dy = a Cos t
dt ⎜⎜ tan t 2 2 ⎟⎟ dt
⎝ 2 ⎠
2 2
ds ⎛ dx ⎞ ⎛ dy ⎞
∴ = ⎜ ⎟ +⎜ ⎟
dt ⎝ dt ⎠ ⎝ dt ⎠
1
⎡ ⎡ Sec 2 t ⎤
2
⎤ 2
⎡ a 2 S int Sec 2 t a 2 Sec 4 t ⎤
⎢ 2⎢
= a − S int + 2 ⎥ + a Cos t ⎥
2 2
, = ⎢a −
2 2 + 2⎥
⎢ ⎢ 2 tan ⎥
t ⎥ ⎢ tan t 4 tan 2 t

⎢⎣ ⎣ 2⎦ ⎥⎦ ⎣ 2 2⎦

= a Cot t

www.bookspar.com | VTU NOTES | QUESTION PAPERS 56 of 124


www.bookspar.com | VTU NOTES | QUESTION PAPERS | NEWS | RESULTS | FORUMS

2011
ds dr
Find and for the following curves
dθ dθ

1. r = a (1 – Cos θ)
2 .r2 = a2 Cos 2θ
3. r = a eθCot α
Solution of 1
r = a (1 – Cos θ)
Differentiating r w.r.t θ we get
dr
= a Sin θ

Hence
1

ds ⎧⎪ 2 ⎛ dr ⎞ ⎫⎪ 2
2

= ⎨r + ⎜ ⎟ ⎬
dθ ⎪⎩ ⎝ dθ ⎠ ⎪⎭
1
= {a2 (1 – Cos θ)2 + a2 Sin2 θ} 2
1
= {a2 (1 – 2 Cos θ + Cos2θ) + a2 Sin2 θ} 2
1
= {a2 – 2a2 Cos θ + a2} 2
1
= {2a2 – 2a2 Cos θ} 2
1
= a{ 2(1 – Cos θ)} 2
1
= a {2 (2 Sin2 θ )} 2
2

= 2 a Sin θ
ds
dθ 2
1

ds ⎧⎪ ⎛ dθ ⎞ ⎫⎪ 2
2

And = ⎨1 + r 2 ⎜ ⎟ ⎬
dr ⎪⎩ ⎝ dr ⎠ ⎪⎭
1
⎧ a (1 − Cosθ ) ⎫
2 2 2
= ⎨1 + ⎬
⎩ a 2 Sin 2θ ⎭

www.bookspar.com | VTU NOTES | QUESTION PAPERS 57 of 124


www.bookspar.com | VTU NOTES | QUESTION PAPERS | NEWS | RESULTS | FORUMS

2011
1
⎧ Sin 2θ + (1 − Cosθ ) 2 ⎫ 2
= ⎨ ⎬
⎩ Sin 2θ ⎭

⎛θ ⎞
2Sin⎜ ⎟ 2Sin θ
{2(1 − Cosθ )}
1

=
2
= ⎝ 2⎠
= 2
Sinθ Sinθ 2 Sin θ Cos θ
2 2
ds 1
=
dr Cos θ
2
Solution of 2
r2 = a2 Cos 2θ
Differentiating W.r.t ‘θ’ we get
dr
2r = -a2 Sin 2θ . 2

dr
r = -a2 Sin 2θ

dr a 2 Sin2 βθ
=
dθ r
Hence
1

ds ⎧⎪ 2 ⎛ dr ⎞ 2 ⎫⎪ 2
= ⎨r + ⎜ ⎟ ⎬
dθ ⎪⎩ ⎝ dθ ⎠ ⎪⎭
1
⎧⎪ ⎛ − a 2 Sin 2θ ⎞
2
⎫⎪ 2
= ⎨r 2 + ⎜⎜ ⎟⎟ ⎬
⎪⎩ ⎝ r ⎠ ⎪⎭
1
⎧ a 4 Sin 2 2θ ⎫ 2
= ⎨r 2 + ⎬
⎩ r2 ⎭
1
⎧ a 4 Sin 2θ ⎫ 2
= ⎨a 2 Cos 2θ + 2 ⎬
⎩ a Cos 2θ ⎭
1
⎧ a 4 Cos 2 2θ + a 4 Sin 2 2θ ⎫ 2
= ⎨ ⎬
⎩ a 2 Cos 2θ ⎭

www.bookspar.com | VTU NOTES | QUESTION PAPERS 58 of 124


www.bookspar.com | VTU NOTES | QUESTION PAPERS | NEWS | RESULTS | FORUMS

2011
1
⎧ a4 ⎫2 a
= ⎨ 2 ⎬ =
⎩ a Cos 2θ ⎭ Cos 2θ
1
ds ⎧ a ⎫2 a2 4
Or = ⎨ 2⎬ = (∵r2 = a2 Cos 2θ)
dθ ⎩ ⎭
r r

1
ds ⎧⎪ ⎛ dθ ⎞ ⎫⎪
2 2

And = ⎨1 + r 2 ⎜ ⎟ ⎬
dr ⎪⎩ ⎝ dr ⎠ ⎪⎭
1
⎧⎪ ⎛ r ⎞ ⎫⎪
2 2

= ⎨1 + r 2 ⎜ ⎟ ⎬
⎪⎩ ⎝ − a Sin 2θ ⎠ ⎪⎭
2

1
⎧ r4 ⎫ 2
= ⎨1 + 4 2 ⎬
⎩ a Sin 2θ ⎭
1
⎧ a 4 Sin 2 2θ + a 4 Cos 2 2θ ⎫ 2
=⎨ ⎬
⎩ a 4 sin 2 2θ ⎭
1
⎧ a4 ⎫ 2 1
=⎨ 4 ⎬ = = Cosec 2θ
⎩ a sin 2θ ⎭ Sin 2θ
2

Solution 3
r = aeθCot α , here α is constant
Differentiating w.r.t ‘θ’ we get
dr
= a eθCot α. Cot α

Hence
1

ds ⎧⎪ 2 ⎛ dr ⎞ 2 ⎫⎪ 2
= ⎨r + ⎜ ⎟ ⎬
dθ ⎪⎩ ⎝ dθ ⎠ ⎪⎭
1
= {a2 e2θCotα + a2 e2θCot α cot2 α} 2

1
=a eθCot α {1 + Cot2α} 2
1

=a eθCot α {Cosce2α} 2

www.bookspar.com | VTU NOTES | QUESTION PAPERS 59 of 124


www.bookspar.com | VTU NOTES | QUESTION PAPERS | NEWS | RESULTS | FORUMS

2011
ds
= a eθCot α Cosce α

1
ds ⎧⎪ 2 ⎛ dθ ⎞ ⎪

2 2

and = ⎨1 + r ⎜ ⎟ ⎬
dr ⎪⎩ ⎝ dr ⎠ ⎪⎭
1
⎧ 1 ⎫ 2
= ⎨1 + a 2 e 2θCotα 2 2θCotαCot 2α ⎬
⎩ a e ⎭
1

= {1 + tan2 α} 2

= {Sec2 α} 2 = Sec α
Exercises:

ds ds
Find and to the following curves.
dr dθ
1. rn = an Cos nθ
2. r (1 + Cos θ) = a
3. rθ = a

Note:

We have Sin φ = r and
ds
dr
Cos φ =
ds
1
dr
∴ = Cosφ = (1 – Sin2φ) 2
ds
1
⎡ p ⎤ 2 2
= ⎢1 − 2 ⎥ Since P = r Sin φ.
⎣ r ⎦

dr r 2 − p2
=
ds r

www.bookspar.com | VTU NOTES | QUESTION PAPERS 60 of 124


www.bookspar.com | VTU NOTES | QUESTION PAPERS | NEWS | RESULTS | FORUMS

2011

ds r
∴ =
dr r − p2
2

r+δr
P(x,y)
φ OR = P
r
O X
P
OR p
R Sin φ = =
OP r

∴ P = r Sin φ

⎛ dθ ⎞
Prove that with usual notations tan φ = r ⎜ ⎟
⎝ dr ⎠

Let P (r, θ) be any point on the curve r = f (θ)



∴ X O P = θ and OP = r
Let PL be the tangent to the curve at P subtending an angle ψ with the positive direction of
the initial line (x – axis) and φ be the angle between the radius vector OP and the tangent PL.

That is O P L = φ
From the figure we have

www.bookspar.com | VTU NOTES | QUESTION PAPERS 61 of 124


www.bookspar.com | VTU NOTES | QUESTION PAPERS | NEWS | RESULTS | FORUMS

2011
ψ = φ+θ
(Recall from geometry that an exterior angle is equal to the sum of the interior opposite angles)
⇒ tan ψ = tan (φ + θ )

tan φ + tan φ
or tan ψ …(1)
1 - tan φ tan θ
Let (x, y) be the Cartesian coordinates of P so that we have,
X = r cos θ , y = r sin θ
Since r is a function of θ , we can as well regard these as parametric equations in terms of θ .
We also know from the geometrical meaning of the derivative that
dy
tan ψ = = slope of the tangent PL
dx
dy dx
ie., tan ψ = since x and y are function of θ
dθ dθ
d
(r sin θ ) r cos θ + r′ sin θ
ie., tan ψ = dθ
dr
= where r′ =
d
(r cos θ ) - r sin θ + r′ cos θ dθ

Dividing both the numerator and denominator by r′ cos θ we have,
r cos θ r′ sin θ
+
tan ψ = r′ cos θ r′ cos θ
− r sin θ r′ cos θ
+
r′ cos θ r′ cos θ
r
+ tan θ
Or tan ψ = r ′ …(2)
r
1 - . tan θ
r′
Comparing equations (1) and (2) we get
r r ⎛ dθ ⎞
tan φ = = or tanφ = r⎜ ⎟
r ′ ⎛ dr ⎞ ⎝ dr ⎠
⎜ ⎟
⎝ dθ ⎠

www.bookspar.com | VTU NOTES | QUESTION PAPERS 62 of 124


www.bookspar.com | VTU NOTES | QUESTION PAPERS | NEWS | RESULTS | FORUMS

2
1 1 1 ⎛ dr ⎞
Prove with usual notations = 2+ 4⎜ ⎟ or
p 2
r r ⎝ dθ ⎠
2
1 ⎛ du ⎞ 1
= u 2 + ⎜ ⎟ where u =
⎝ dθ ⎠
2
p r

Proof :

Let O be the pole and OL be the initial line. Let P (r, θ ) be any point on the curve and hence

we have OP = r and L O P = θ

Draw ON = p (say) perpendicular from the pole on the tangent at P and let φ be the angle
made by the radius vector with the tangent.
∧ ∧
From the figure O N P = 90 L O P = θ
Now from the right angled triangle ONP
ON
sin φ =
OP
P
ie., sin φ = or p = r sin φ
r
we have p = r sin φ …(1)
1 dr
and cot φ = …(2)
r dθ
Squaring equation (1) and taking the reciprocal we get,

www.bookspar.com | VTU NOTES | QUESTION PAPERS 63 of 124


www.bookspar.com | VTU NOTES | QUESTION PAPERS | NEWS | RESULTS | FORUMS

2011
1 1 1 1 1
= 2 . ie., = 2 cosec 2 φ
p 2
r sin 2 φ p 2
r

Or
1
p 2
=
1
r 2
(
1 + cot 2 φ )
Now using (2) we get,

1 1 ⎡ 1 ⎛ dr ⎞
2⎤
= 2 ⎢1 + 2 ⎜ ⎟ ⎥
⎝ dθ ⎠
2
p r ⎢⎣ r ⎥⎦
2
1 1 1 ⎛ dr ⎞
Or = 2 + 4 ⎜ ⎟ …(3)
⎝ dθ ⎠
2
p r r
1
Further, let =u
r
Differentiating w.r.t. θ we get,
2 2
1 ⎛ dr ⎞ du 1 ⎛ dr ⎞ ⎛ du ⎞
− 2 ⎜ ⎟= ⇒ 4 ⎜ ⎟ = ⎜ ⎟ , by squaring
r ⎝ dθ ⎠ dθ r ⎝ dθ ⎠ ⎝ dθ ⎠
Thus (3) now becomes
2
1 ⎛ du ⎞
= u2 + ⎜ ⎟ …(4)
⎝ dθ ⎠
2
p

1. Find the angle of intersection of the curves:


r = a (1 + cos θ ) & r = b (1 - cos θ )

Solution : r = a (1 + cos θ) : r = b(1 – cos θ)


⇒ log r = log a + log (1 + cos θ) : log r = log b + log (1 – cos θ)

Differentiating these w.r.t. θ we get


1 dr - sin θ 1 dr sin θ
= 0 + : = 0 +
r dθ 1 + cos θ r dθ 1 − cos θ
− 2 sin (θ/2 ) cos (θ/2 ) 2 sin (θ/2 ) cos (θ/2 )
cot φ1 = : cot φ 2 =
2 cos 2 (θ/2 ) 2 sin 2 (θ/2 )
ie., cot φ1 = - tan (θ /2 ) = cot (π/2 + θ /2 ) : cot φ 2 = cot (θ /2 )

⇒ φ 1 = π /2 + θ /2 : φ 2 = θ/2

www.bookspar.com | VTU NOTES | QUESTION PAPERS 64 of 124


www.bookspar.com | VTU NOTES | QUESTION PAPERS | NEWS | RESULTS | FORUMS

2011
∴ angle of intersection = φ1 - φ 2 = π/2 + θ/2 - θ/2 = π/2

Hence the curves intersect orthogonally.


2. S.T. the curves r = a (1 + sin θ ) & r = a (1 - sin θ ) cut each other orthogonally

Solution : log r = log a + log (1 + sin θ) : log r = log a + log (1 – sin θ)


Differentiating these w.r.t θ we get
1 dr cos θ 1 dr - cos θ
= : =
r dθ 1 + sin θ r dθ 1 − sin θ
cos θ - cos θ
ie., cot φ1 = : cot φ 2 =
1 + sin θ 1 − sin θ
1 + sin θ 1 - sin θ
We have tan φ1 = and tan φ 2 =
cos θ - cos θ

1 - sin 2 θ cos 2 θ
∴ tan φ1 . tan φ 2 = = = -1
− cos 2 θ - cos 2 θ
Hence the curves intersect orthogonally.

3. Find the angle of intersection of the curves:


r = sin θ + cos θ, r = 2 sin θ
Solution : log r = log (sin θ + cos θ) : r = 2 sin θ
⇒ log r = log (sin θ + cos θ ) : log r = log 2 + log (sin θ)

Differentiating these w.r.t θ we get


1 dr cos θ - sin θ 1 dr cos θ
= : =
r dθ sin θ + cos θ r dθ sin θ
cos θ (1 - tan θ )
ie., cot φ1 = : cot φ 2 = cot θ ⇒ φ 2 = θ
cos θ (1 + tan θ )
ie., cot φ1 = cot (π/4 + θ ) ⇒ φ1 = π/4 + θ

∴ φ1 - φ 2 = π/4 + θ - θ = π/4

The angle of intersection is π / 4

www.bookspar.com | VTU NOTES | QUESTION PAPERS 65 of 124


www.bookspar.com | VTU NOTES | QUESTION PAPERS | NEWS | RESULTS | FORUMS

2011

4. Find the angle of the curves: r = a log θ and r = a/ log θ


Solution : r = a log θ : r = a/ log θ
⇒ log r = log a + log (log θ ) : log r = log a - log (log θ )
Differentiating these w.r.t q, we get,
1 dr 1 1 dr 1
= : =−
r dθ log θ . θ r dθ log θ . θ
1 1
ie., cot φ1 = : cot φ 2 = -
θ log θ θ log θ
Note : We can not find φ1 and φ 2 explicitly.
∴ tan φ1 = θ log θ : tan φ 2 = θ log θ

Now consider, tan φ1 = θ log θ : tan φ 2 = - θ log θ

2 θ log θ 2θ log θ
Now consider, tan (φ1 - φ 2 ) = = ......…(1)
1 + tan φ1 tan φ 2 1 − (θ log θ)2

We have to find θ by solving the given pair of equations :


r = a log θ and r = a/log θ
a
Equating the R.H.S we have a log θ =
log θ

ie., (log θ = 1 )2 = 1 or log θ ⇒ θ = e


Substituting θ = e in (1) we get

tan (φ1 − φ 2 ) =
2e
(∵ log e = 1)
1 - e2
⎛ 2e ⎞
∴ angle of intersection φ1 - φ 2 = tan -1 ⎜ 2 ⎟
= 2 tan -1 e
⎝1- e ⎠
5. Find the angle of intersection of the curves:
r = a (1 – cos θ ) and r = 2a cos θ
Solution : r = a (1 – cos θ) : r = 2a cos θ
Taking logarithms we have,
Log r = log a + log (1 – cos θ) : log r = log 2a + log (cos θ)
Differentiating these w.r.t θ, we get,

www.bookspar.com | VTU NOTES | QUESTION PAPERS 66 of 124


www.bookspar.com | VTU NOTES | QUESTION PAPERS | NEWS | RESULTS | FORUMS

2011
1 dr sin θ 1 dr - sin θ
= : =
r dθ 1 - cos θ r dθ cos θ
2 sin (θ/2 ) cos (θ/2 )
ie., cot φ1 = : cot φ 2 = - tan θ
2 sin 2 (θ/2 )
ie., cot φ1 = cot (θ /2 ) : cot φ 2 = cot (π/2 + θ )

⇒ φ1 = θ /2 : φ 2 = π/2 + θ

∴ φ1 − φ 2 = θ/2 - π/2 - θ = π / 2 +θ 2 …(1)

Now consider r = a (1 - cos θ ) = 2a cos θ

Or 3 cos θ = 1 or θ = cos -1 (1/3)


Substituting this value in (1) we get,
The angle of intersection π / 2 + 1/2 . cos -1 (1/3)

6. Find the angle of intersection of the curves :


r = aθ and r = a / θ
Solution : r = aθ : r=a/θ
⇒ log r = log a + log θ : log r = log a - log θ
Differentiating these w.r.t θ, we get,
1 dr 1 1 dr 1
= : =-
r dθ θ r dθ θ
1 1
ie., cot φ1 = : cot φ 2 = -
θ θ
or tan φ1 = θ : tan φ 2 = - θ

Also by equating the R.H.S of the given equations we have


aθ = a/ θ or θ 2 = 1 ⇒ θ = ± 1
When θ = 1, tan φ1 = 1, tan φ 2 = - 1 and
When θ = - 1, tan φ1 = - 1, tan φ 2 = 1 .
∴ tan φ1 . tan φ 2 = - 1 ⇒ φ1 - φ 2 = π /2

The curves intersect at right angles.

www.bookspar.com | VTU NOTES | QUESTION PAPERS 67 of 124


www.bookspar.com | VTU NOTES | QUESTION PAPERS | NEWS | RESULTS | FORUMS

2011

7. Find the pedal equation of the curve: r (1 - cos θ ) = 2a


Solution : r (1 - cos θ ) = 2a
⇒ log r + log (1 - cos θ ) = log 2a

Differentiating w.r.t θ, we get


1 dr sin θ 1 dr - sin θ
+ = 0 or =
r dθ 1 - cos θ r dθ 1 - cos θ
- 2 sin (θ/2 ) cos (θ/2 )
∴ cot φ = = - cot (θ/2 )
2 sin 2 (θ/2 )
ie., ∴ cot φ = cot (- θ /2 ) ⇒ φ = - (θ /2 )
Consider p = r sin φ
∴ p = r sin (- θ /2 ) or p = - r sin (θ /2 )

Now we have, r (1 - cos θ ) = 2a …(1)


p = - r sin (θ /2 ) …(2)
We have to eliminate θ from (1) and (2)
(1) can be put in form r . 2 sin 2 (θ / 2 ) = 2a

ie., r sin 2 (θ/2 ) = a


But p/-r = sin (θ/2), from (2)
⎛ p2 ⎞
∴ r ⎜⎜ 2 ⎟⎟ = a or p 2 = ar
⎝r ⎠
Thus p2 = ar is the required pedal equation.

8. Find the pedal equation of the curve: r2 = a2 sec 2θ


Solution : r2 = a2 sec 2θ
⇒ 2 log r = 2 log a + log (sec 2θ)
Differentiating w.r.t θ , we get,
2 dr 2 sec 2θ tan 2θ 1 dr
= ie., = tan 2θ
r dθ sec 2θ r dθ
ie., cot φ = cot (π /2 - 2θ ) ⇒ φ = π/2 - 2θ

www.bookspar.com | VTU NOTES | QUESTION PAPERS 68 of 124


www.bookspar.com | VTU NOTES | QUESTION PAPERS | NEWS | RESULTS | FORUMS

2011
Consider p = r sin φ ∴ p = r sin (π /2 - 2θ ) ie., p = r cos 2 θ

Now we have, r 2 = a 2 sec 2θ …(1)


p = r cos 2θ …(2)
From (2) p/r = cos 2θ or r/p = sec 2 θ
Substituting in (1) we get, r 2 = a 2 (r/p ) or pr = a2
Thus pr = a2 is the required pedal equation.

9. Find the pedal equation of the curve: rn = an cos nθ


Solution : rn = an cos nθ
⇒ n log r = n log a + log (cos n θ )

Differentiating w.r.t q we get


n dr - n sin nθ 1 dr
= ie., = - tan nθ
r dθ cos nθ r dθ
∴ cot φ = cot (π /2 + n θ ) ⇒ φ = π /2 + n θ

Consider p = r sin φ
∴ p = r sin (π /2 = nθ ) ie., p = r cos nθ

Now we have, r n = a n cos nθ …(1)


p = r cos nθ …(2)

∴ (1) as a consequenc e of (2) is r n = a n (p/r )


Thus rn + 1 = pan is the required pedal equation.

10. Find the pedal equation of the curve: rm = am (cos mθ + sin mθ)
Solution : rm = am (cos mθ + sin mθ)
Differentiating w.r.t θ, we get,
m dr - m sin mθ + m cos mθ
=
r dθ cos mθ + sin mθ
1 dr cos mθ - sin mθ cos mθ (1 - tan mθ )
ie., = =
r dθ cos mθ + sin mθ cos mθ (1 + tan mθ )
∴ cot φ = cot (π /4 + m θ ) ⇒ φ = π /4 + m θ

www.bookspar.com | VTU NOTES | QUESTION PAPERS 69 of 124


www.bookspar.com | VTU NOTES | QUESTION PAPERS | NEWS | RESULTS | FORUMS

2011
Consider p = r sin φ
∴ p = r sin (π /4 + m θ )

ie., p = r [sin (π /4 ) cos m θ + cos (π/4 ) sin m θ ]

ie., p =
r
(cos mθ + sin mθ )
2
(we have used the formula of sin (A + B) and also the values sin (π/4 ) = cos (π/4 ) = 1/ 2 )
Now we have, r m = a m (cos m θ + sin m θ ) …(1)

p=
r
(cos m θ + sin mθ ) …(2)
2
Using (2) in (1) we get,
p 2
rm = am . or r m +1 = 2 a m p
r
Thus r m + 1 = 2 a m p is the required pedal equation.

11. Establish the pedal equation of the curve:


( )
r n = a n sin n θ + b n cos nθ in the form p 2 a 2n + b 2 n = r 2 n + 2

Solution : We have r n = a n sin nθ + b n cos nθ


(
⇒ n log r = log a n sin nθ + b n cos nθ )
Differentiating w.r.t θ we get

n dr na n cos nθ - nb n sin nθ
= n
r dθ a sin nθ + b n cos nθ
a n cos nθ - b n sin nθ
Dividing by n, cot φ = n
a sin nθ + b n cos nθ
Consider p = r sin φ
Since φ cannot be found, squaring and taking the reciprocal we get,
1
p 2
1 1 1
(
= 2 cosec 2 φ or 2 = 2 1 + cot 2 φ
r p r
)
1 1
∴ 2= 2
⎧⎪ (
⎨1 + n
2
)
a n cos nθ - b n sin nθ ⎫⎪
2⎬
p r ⎪⎩ (a sin nθ + b n cos nθ ⎪⎭ )

www.bookspar.com | VTU NOTES | QUESTION PAPERS 70 of 124


www.bookspar.com | VTU NOTES | QUESTION PAPERS | NEWS | RESULTS | FORUMS

2011

1 1
ie., 2 = 2 ⎨
( ) (
⎧⎪ a n sin nθ + b n cos nθ 2 + a n cos nθ - b n sin nθ ) ⎫⎪⎬
2

p r ⎪⎩ (
a n sin nθ + b n cos nθ
2
) ⎪⎭

1 1
ie., 2 = 2 ⎨
( ) (
⎧⎪ a 2 n sin 2 nθ + cos 2 nθ + b 2 n cos 2 nθ + sin 2 nθ ⎫⎪

)
p r ⎪⎩ (
a n sin nθ + b n cos nθ
2
) ⎪⎭

(product terms cancels out in the numerator)


1 1 a 2 n + b2 n
=
( )
ie., .
p 2 r 2 a n sin nθ + b n cos nθ 2

1 1 a 2 n + b 2n
= , by using the given equation.
( )
or .
p2 r 2 rn
2

(
∴ p2 a 2 n + b2 n )= r 2 n+2
is the required pedal equation.

12. Define Curvature and Radius of curvature

Solution: A Curve Cuts at every point on it. Which is determined by the tangent drawn.

Y y = f (x) Tangent

s P (x,y)
A

O ψ X
Let P be a point on the curve y= f (x) at the length ‘s’ from a fixed point A on it. Let the tangent
at ‘P’ makes are angle ψ with positive direction of x – axis. As the point ‘P’ moves along curve, both s
and ψ vary.

The rate of change ψ w.r.t s, i.e., as called the Curvature of the curve at ‘P’.
ds
The reciprocal of the Curvature at P is called the radius of curvature at P and is denoted by ρ.

1 ds
∴ρ = =
dψ dψ
ds

www.bookspar.com | VTU NOTES | QUESTION PAPERS 71 of 124


www.bookspar.com | VTU NOTES | QUESTION PAPERS | NEWS | RESULTS | FORUMS

2011
ds 1 dψ
∴ρ = (or) =
dψ ρ ds
1 dψ
Also denoted ρ = ∴K =
K ds
K is read it as Kappa.

13. Derive an expression for radius of curvature in Cartesian form.


Solution :(1) Cartesian Form:

Y = f (x)

P T
A S ψ
dy
ψ
ψ P dx R
X
O

Let y = f (x) be the curve in Cartesian form.


dy
We know that, tan ψ = (From Figure) ----------- (1)
dx
Where ψ is the angle made by the tangent at P with x – axis. Differentiating (1) W.r.t x, we get

dψ d2y
Sec2ψ . =
dx dx 2
d2y dψ
i.e., 2
= Sec2 ψ
dx dx
d ψ
= (1 + tan2 ψ)
dx
d ψ ds
= (1 + tan2 ψ) .
dx dx
1

1 ⎧⎪ ⎛ dy ⎞ ⎫⎪ 2
2

= (1 + tan ψ)
2
⎨1 + ⎜ ⎟ ⎬
ρ ⎪⎩ ⎝ dx ⎠ ⎪⎭
1
⎧⎪ ⎛ dy ⎞ 2 ⎫⎪ 1 ⎧⎪ ⎛ dy ⎞ 2 ⎫⎪ 2
= ⎨1 + ⎜ ⎟ ⎬ . . ⎨1 + ⎜ ⎟ ⎬ from eqn (1)
⎪⎩ ⎝ dx ⎠ ⎪⎭ ρ ⎪⎩ ⎝ dx ⎠ ⎪⎭

www.bookspar.com | VTU NOTES | QUESTION PAPERS 72 of 124


www.bookspar.com | VTU NOTES | QUESTION PAPERS | NEWS | RESULTS | FORUMS

2011
3

1 ⎧⎪ ⎛ dy ⎞ 2 ⎫⎪ 2
= ⎨1 + ⎜ ⎟ ⎬
ρ ⎪⎩ ⎝ dx ⎠ ⎪⎭
3
⎡ ⎛ dy ⎞ 2 ⎤ 2
⎢1 + ⎜ ⎟ ⎥
⎢ ⎝ dx ⎠ ⎦⎥
∴ρ= ⎣ 2
d y
dx 2
∴ ρ = ( 1+ y1 2 )3/2 ……………(1)
y2
dy d2y
Where y1 = , y2 =
dx dx 2
This is the formula for Radius of Curvature in Cartesian Form.
14. Show that the Curvature of a Circle at any point on it, is a Constant

Tangent

O r PP P

A ψ X
T

Solution: Consider a Circle of radius r. Let A be a fixed point and ‘P’ be a given point on the circle
such that arc AP = S.
Let the angle between the tangent to the Circle at A and P be ψ. Then clearly AOP = ψ.

∴ AP = rψ
i.e., S = rψ
This is the intrinsic equation of the circle.
Differentiating w.r.t S we get

dψ dψ 1
1=r Or K = =
ds ds r
1
∴ K = which is Constant
r

Hence the Curvature of the Circle at any point on it is constant.

www.bookspar.com | VTU NOTES | QUESTION PAPERS 73 of 124


www.bookspar.com | VTU NOTES | QUESTION PAPERS | NEWS | RESULTS | FORUMS

2011

15. Derive an expression for radius of curvature in parametric form.

Solution: We have ρ = (1 + y 12 )3/2

y2
Let x =f (t), y = g (t) be the curve in Parametric Form.
dy •
dy dt Y
Then y1 = = = •
dx dx
dt X
⎛ • ⎞ ⎛ • ⎞
d2y d ⎛ dy ⎞ d ⎜ Y ⎟ d ⎜Y ⎟ dt
Y2 = = ⎜ ⎟ =
dx 2 dx ⎝ dx ⎠ dx ⎜⎜ X• ⎟⎟ dt ⎜⎜ X• ⎟⎟ dx
⎝ ⎠ ⎝ ⎠
⎛ ⎞
⎛ • ⎞ ⎛ • ⎞
d ⎜ Y ⎟ ⎜⎜ 1 ⎟⎟ d ⎜ Y ⎟ 1
= . .
dt ⎜⎜ X• ⎟⎟ ⎜ dx ⎟ dt ⎜⎜ X• ⎟⎟ X•
⎝ ⎠ ⎜ ⎟ ⎝ ⎠
⎝ dt ⎠
xy − yx 1
=
(x )2 ( X• )

X y − yx
∴ Y2 = 3
⎛•⎞
⎜ x⎟
⎝ ⎠
Substituting Y1 and Y2 in equation 1.

ρ=
(1 + Y ) 1
2
3
2
, we get
y2

( ( )
3
⎡1+ Y 2⎤ 2

⎢ x ⎥⎦
ρ= ⎣
xy − yx
(x )3
ρ=
[(x) + ( y ) ]
2 2
3
2
-------------------- (2)
xy − yx
Equation (2) is called the Radius of Curvature in Parametric Form.

www.bookspar.com | VTU NOTES | QUESTION PAPERS 74 of 124


www.bookspar.com | VTU NOTES | QUESTION PAPERS | NEWS | RESULTS | FORUMS

2011

16. Derive an expression for radius of curvature in polar form.


Solution: Let r = f (θ) be the curve in the Polar Form. We know that, Angle between the
tangent and radius vector,


Tan φ = r
dr
1
= r.
dr

r
i.e., tan φ =
(dr dθ )
Differentiate w.r.t ‘θ’ we get

dr dr d 2r
. −r

= dθ dθ 2 dθ
2
Sec2 φ .
dθ ⎛ dr ⎞
⎜ ⎟
⎝ dθ ⎠


⎧ dr
1 ⎪ dθ r
2
(
d 2r 2 ⎫
dθ ⎪
)
∴ = 2 ⎨ ⎬
dθ Sec φ ⎪

dr

2


( )

1 (
⎧ dr
⎪ dθ
2
)
− r d 2r 2 ⎫
dθ ⎪
= ⎨ ⎬
1 + tan φ ⎪
2


dr (dθ
2
) ⎪

⎧ ⎛ dr ⎞ 2
d 2r ⎫
⎪⎜ ⎟ −r ⎪
1 ⎪ ⎝ dθ ⎠ dθ 2 ⎪
= ⎨ ⎬
r2 ⎪ ⎛ dr ⎞
2

1+ ⎜ ⎟
⎛ dr ⎞ ⎩
2
⎪ ⎝ dθ ⎠ ⎪⎭
⎜ ⎟
⎝ dθ ⎠
2
⎛ dr ⎞ d 2r
⎜ ⎟ −r 2
dφ dθ ⎠ dθ
= ⎝
dθ ⎛ dr ⎞
2

⎜ ⎟ +r
2

⎝ d θ ⎠

From figure ψ = θ + φ

Differentiating w.r.t θ, we get

www.bookspar.com | VTU NOTES | QUESTION PAPERS 75 of 124


www.bookspar.com | VTU NOTES | QUESTION PAPERS | NEWS | RESULTS | FORUMS

2011
dψ dφ
=1+
dθ dθ

2
⎛ dr ⎞ d 2r
⎜ ⎟ −r
dψ dθ ⎠ dθ 2
∴ =1+ ⎝
dθ ⎛ dr ⎞
2

⎜ ⎟ +r
2

⎝ dθ ⎠
2
⎛ dr ⎞ d 2r
r 2 + 2⎜ ⎟ −r
dψ ⎝ dθ ⎠ dθ 2
=
dθ ⎛ dr ⎞
2

⎜ ⎟ +r
2

⎝ d θ ⎠
1

ds ⎧⎪ ⎛ dr ⎞ ⎫⎪ 2
2

Also we know that = ⎨r 2 + ⎜ ⎟ ⎬


dθ ⎪⎩ ⎝ dθ ⎠ ⎪⎭

ds ds dθ
Now, ρ= = .
dψ dθ dψ
2
⎛ dr ⎞
⎜ ⎟ +r
1 2
⎧⎪ 2 ⎛ dr ⎞ ⎫⎪
2 2
⎝ d θ ⎠
= ⎨r + ⎜ ⎟ ⎬ .
⎪⎩ ⎝ dθ ⎠ ⎪⎭
2
⎛ dr ⎞ d 2r
r + 2⎜
2
⎟ −r
⎝ dθ ⎠ dθ 2

ρ = 2
(r
+ r12 ) 2
2
3

----------- (3)
r + 2r12 − rr2

dr d 2r
Where r1 = , r2 =
dθ dθ 2

Equation (3) as called the radius of curvature in Polar form.

17. Derive an expression for radius of curvature in pedal form.


Solution: Let p = r Sin φ be the curve in Polar Form.

We have p = r Sin φ

Differentiating p W.r.t r, we get

dp dφ
= Sin φ + r Cos φ
dr dr

www.bookspar.com | VTU NOTES | QUESTION PAPERS 76 of 124


www.bookspar.com | VTU NOTES | QUESTION PAPERS | NEWS | RESULTS | FORUMS

2011

dθ dr
But Sin φ = r , Cos φ=
ds ds

dp dθ dr dφ
∴ =r +r
dr ds ds dr

dθ dφ dr
=r + .
ds dr ds

dθ dφ
=r +r
ds ds

d
=r (θ + φ)
ds


=r
ds

dp dψ
∴ =r
dr ds

dp 1 dψ 1
= r. ∵ =
dr ρ ds ρ

dr
∴ ρ= r --------------- (4)
dp

Equation (4) is called Radius of Curvature of the Curve in Polar Form.

18. Find the radius of curvature at (x, y) for the curve ay2 = x3.

Solution: Given ay2 = x3 -------- (1) is in Cartesian form.

We have, Radius of curvature in Cartesian form.

ρ=
(1 + y )2
1
3
2
------------ (2)
y2

Differentiating (1) w.r.t x we get,

www.bookspar.com | VTU NOTES | QUESTION PAPERS 77 of 124


www.bookspar.com | VTU NOTES | QUESTION PAPERS | NEWS | RESULTS | FORUMS

2011

dy dy 3 x2 3x 2 3 x
a 2y = 3x 2 ⇒ = y1 = = 1
=
dx dx 2 ay 2 a
⎛ x3 ⎞ 2
2a⎜⎜ ⎟⎟
⎝ a ⎠
3 x
∴ y1 =
2 a

Differentiating y1 w.r.t ‘x’ we get

d2y 3 1 3
y2 = 2
= =
dx 2 a 2 x 4 a x

Substitute Y1 and Y2 in (2), we get.


x (4a + 9 x )
3
2
ρ=
6a

⎛ x⎞
19. Find the radius of curvature at (x,y) for the curve y = c log Sec ⎜ ⎟
⎝c⎠
⎛ x⎞
Solution: Given y = c log Sec ⎜ ⎟ -------- (1)
⎝c⎠
Differentiating (1) w.r.t x, we get

1 ⎛ x⎞ ⎛ x⎞ ⎛ x⎞ 1 ⎛ x⎞
y1 = c. .Sec⎜ ⎟ tan⎜ ⎟ = c tan ⎜ ⎟ . = tan ⎜ ⎟
⎛ x⎞ ⎝c⎠ ⎝c⎠ ⎝c⎠ c ⎝c⎠
Sec⎜ ⎟
⎝c⎠
Differentiating y1 W.r.t x we get

⎛ x⎞ 1 1 ⎛ x⎞
y 2 = Sec 2 ⎜ ⎟ = Sec 2 ⎜ ⎟
⎝c⎠ c c ⎝c⎠

Substitute y1 and y2 in δ =
(1 + y ) 2
1
3
2

y2

∴δ =
[1 + tan (x c )] = [Sec (x c )]
2
3
2 2
3
2

Sec (x ) Sec (x )
1 2 1 2

c c c c

( c)
ρ = cSec x

www.bookspar.com | VTU NOTES | QUESTION PAPERS 78 of 124


www.bookspar.com | VTU NOTES | QUESTION PAPERS | NEWS | RESULTS | FORUMS

2011

20. Find the radius of curvature at the point ‘t’ on the curve x = a (t + Sint), y = a (1 – Cost).

Solution: Given Curves are in Parametric Form

∴ Radius of Curvature, ρ =
((x) + (y) 2
2

------ (1)
2
)
3

xy − yx
Differentiating the given Curves W.r.t t, we get

dx dy
x= = a (1 + Cost) y = = a Sint
dt dt
Differentiating W.r.t ‘t’ we get

x = - a Sint, y = a Cost

Substitute x , y , y and x in (1), we get

ρ=
(a (1 + Cost )
2
+ a 2 S int 22
) 3

a(1 + Cost )aCost − aS int( − aS int)

a 3 {1 + 2Cost + Cos 2 t + Sin 2 t } 2


3

=
a 2 {Cost + Cos 2 t + Sin 2 t}

{ }= { } 3

a 2(1 + Cost )
3
2 a 2.2Cos 2 t 2
8a.Cos 3 t
= 2 = 2
(1 + Cost ) 2Cos t
2
2Cos t 2
2 2
ρ = 4aCos t 2
21. Find the Radius of Curvature to x = a Cost + log tan t { ( 2 )}, y = a Sint at t.
{
Solution: Here x = a Cost + log tan t ( 2 )}, y = a Sin t
dx ⎧
⎪ 1 1 ⎫⎪
= a ⎨− S int + 2 t
2 2⎬
.Sec .
dt ⎪⎩ tan t ⎪⎭
2
1
=a { -sint + }
2 sin t / 2 cos t / 2
1
=a { -sint + }
2 sin t / 2
= a { (1-sin 2t) / sint }

dx
= a Cos2t / sint
dt

www.bookspar.com | VTU NOTES | QUESTION PAPERS 79 of 124


www.bookspar.com | VTU NOTES | QUESTION PAPERS | NEWS | RESULTS | FORUMS

2011
dy
and = aCost
dx

dy dy / dt aCost
∴ = = = tant
dx dx / dt a cos t cos t / sin t

dy
∴ = tan t
dx
Differentiating W.r.t ‘x’ we get

d2y dt 1 1
2
= Sec 2 t = Sec 2 t = Sec 2 t.
dx dx dx aCos t
2
S int
dt
d2y 1
2
= = Sec 4 tS int
dx a

Substitute
dy
&
d2y
in ρ =
(1 + y12 ) 3
2
, we get
dx dx 2 y2

i.e., ρ=
(1 + tan t ) 2
3
2
=
aSec3t
=a
Cost
= aCott
4
1 Sec tS int S int
Sec 4tS int
a
∴ρ = a Cot t.

⎛a a⎞
22. Find the Radius of Curvature to x + y = a at ⎜ , ⎟
⎝ 4 4⎠
Solution: Given x+ y= a -------------- (1)

Differentiating (1) w.r.t to ‘x’ we get

1 1 dy
+ =0
2 x 2 y dx

i.e., y1 =
dy
=−
y
=−
( a− x ) (From (1))
dx x x

a
y1 = 1 − ----------- (2)
x

www.bookspar.com | VTU NOTES | QUESTION PAPERS 80 of 124


www.bookspar.com | VTU NOTES | QUESTION PAPERS | NEWS | RESULTS | FORUMS

2011

d2y ⎛ 1 ⎞ −3 a
Also y 2 = = − a ⎜ − ⎟x 2 = 3 -------------- (3)
⎝ 2⎠
2
dx 2x 2

⎛a a⎞
At the given point ⎜ , ⎟
⎝ 4 4⎠
a a 4
Then y1 = 1 − = - 1 & y2 = 1 − =
a
2 2a 2
4
3
a
( )
(1 + y )
3
2 2
∴ Substitute y1 and y2 in ρ= 1

y2

ρ=
(1 + (−1) ) 2
3
2
=
3
2 2 a 2 2a
= =
a
4 4 4 2
a
a
ρ=
2

23. Show that for the Cardioids r = a (1 + Cosθ), ρ2 / r 2 is a constant

Solution: r = a (1 + Cosθ)

dr
= - a Sinθ

We have,

2
1 1 1 ⎛ dr ⎞
= 2 + 4⎜ ⎟ , is Pedal Equation.
P 2
r r ⎝ dθ ⎠

1 1
= 2
+ 4 a2Sin2θ
r r

r 2 + a 2 Sin 2θ a 2 (1 + Cosθ ) 2 + a 2 Sin 2θ


= =
r4 r4

2a 2 (1 + Cosθ )
=
r4

www.bookspar.com | VTU NOTES | QUESTION PAPERS 81 of 124


www.bookspar.com | VTU NOTES | QUESTION PAPERS | NEWS | RESULTS | FORUMS

2011
1 2a 2 ⎛r⎞
∴ = ⎜ ⎟ (∵ r = a (1 + Cosθ)
P2 r4 ⎝a⎠
r
∴ = 1 + Cosθ
a
1 2a
2
= 3
P r

r3
∴ P2 =
2a

Differentiating w.r.t ‘P’ we get


1 2 dr
2P = 3r
2a dp

dr 4ap
=
dp 3r 2

dr 4ap
Now, ρ=r =
dp 3r

ρ2 1 ⎡16a 2 p 2 ⎤ 16a 2 r 3 8a
And = ⎢ ⎥= . =
r2 r ⎣ 9r 2 ⎦ 9r 3 2a 9

1 1 1 r2
24. Find the Radius of Curvature of the Curve = + −
P 2 a 2 b 2 a 2b 2
1 1 1 r2
Solution: Given = + −
P 2 a 2 b 2 a 2b 2

Differentiating w.r.t to P, we get

−2 −1 dr
3
= 2 2 2r
P a b dp

dr a 2 b 2
∴ = 3
dp p r

dr a 2b 2 a 2b 2
∴ ρ = r. = r. 3 = 3
dp pr p

www.bookspar.com | VTU NOTES | QUESTION PAPERS 82 of 124


www.bookspar.com | VTU NOTES | QUESTION PAPERS | NEWS | RESULTS | FORUMS

2011

a
25. Find the Radius of Curvature at (r,θ) on r =
θ
a
Solution: Given r =
θ

Differentiating w.r.t to ‘θ’ we get

dr − a − a 1 − r
= = . =
dθ θ 2 θ θ θ

dr − r
=
dθ θ
2
1 1 1 ⎛ dr ⎞
We have 2 = 2 + 4 ⎜ ⎟
P r r ⎝ dθ ⎠
1 1 r2 1 1
= 2 + 4 2 = 2 + 2 2
r r θ r rθ

1 1 ⎛ 1 ⎞ 1 ⎛ θ 2 + 1⎞
= ⎜1 + ⎟ = ⎜ ⎟
P 2 r 2 ⎝ θ 2 ⎠ r 2 ⎜⎝ θ 2 ⎟⎠


∴P =
θ 2 +1
rθ r. a a.r
P= = r =
θ 2 +1 a2 a2 + r 2
+ 1
r2
Differentiating above result w.r.t to‘P’ we get

dr 1 dr
a 2 + r 2 .a − r.a 2/ r
dp 2/ a 2 + r 2 dp
1=
(
a +r
2 2
)
⎛ 2 r 2a ⎞ dr
2

a +r =⎜ a +r .a−
2 2

⎟ dp
⎝ a2 + r 2 ⎠

(a 2 + r 2 ). a − r 2 a dr
a2 + r 2 =
a2 + r 2 dp

www.bookspar.com | VTU NOTES | QUESTION PAPERS 83 of 124


www.bookspar.com | VTU NOTES | QUESTION PAPERS | NEWS | RESULTS | FORUMS

(a 2
+ r 2 ) a2 + r 2 = a3
dr
dp
(a 2
+ r2 )
3
2
=
dr
a3 dp

Thus, ρ = r.
dr r. a 2 + r 2
=
( )
3
2

dp a3

∴ ρ = 3 (a 2 + r 2 ) 2
r 3

Exercises:
⎛π ψ ⎞
(1) Find the Radius of the Curvature at the point (s, ψ) on S = a log tan ⎜ + ⎟
⎝4 2⎠

(2) Find the Radius of the Curvature of xy = C2 at (x,y)

(3) Find the Radius of the Curvature of xy3 = a4 at (a,a)

(4) Find the Radius of Curvature at the point θ on x = C Sin 2θ (1 + Cos 2θ),
y = C Cos 2θ (1 – Cos 2θ)

(5) If ρ1 and ρ2 are the radii of curvature at the extremities of any chord of the cardiode
16a 2
r = a (1 + Cosθ) which posses through the Pole prove that ρ1 + ρ 2 =
2 2

www.bookspar.com | VTU NOTES | QUESTION PAPERS 84 of 124


www.bookspar.com | VTU NOTES | QUESTION PAPERS | NEWS | RESULTS | FORUMS

2011

DIFFERENTIAL CALCULUS – III

PARTIAL DIFFERENTIATION
Introduction :-
Partial differential equations abound in all branches of science and engineering and
many areas of business. The number of applications is endless.
Partial derivatives have many important uses in math and science. We shall see
that a partial derivative is not much more or less than a particular sort of directional
derivative. The only trick is to have a reliable way of specifying directions ... so most of
this note is concerned with formalizing the idea of direction
So far, we had been dealing with functions of a single independent variable. We will now
consider functions which depend on more than one independent variable; Such
functions are called functions of several variables.
Geometrical Meaning
Suppose the graph of z = f (x,y) is the
surface shown. Consider the partial
derivative of f with respect to x at a point
(x0, y0). Holding y constant and varying x,
we trace out a curve that is the intersection
of the surface with the vertical plane y = y0.
The partial derivative fx(x0,y0). measures
the change in z per unit increase in x along
this curve. That is, fx(x0, y0) is just the slope
of the curve at (x0, y0). The geometrical
interpretation of fy(x0, y0). is analogous.

www.bookspar.com | VTU NOTES | QUESTION PAPERS 85 of 124


www.bookspar.com | VTU NOTES | QUESTION PAPERS | NEWS | RESULTS | FORUMS

2011
Real-World Applications:
Rates of Change:
In the Java applet we saw how the concept of partial derivative could be applied
geometrically to find the slope of the surface in the x and y directions. In the following
two examples we present partial derivatives as rates of change. Specifically we explore
an application to a temperature function ( this example does have a geometric aspect in
terms of the physical model itself) and a second application to electrical circuits, where
no geometry is involved.
I. Temperature on a Metal Plate
The screen capture below shows a current website illustrating thermal flow for chemical
engineering. Our first application will deal with a similar flat plate where temperature
varies with position.
* The example following the picture below is taken from the current text in SM221,223:
Multivariable Calculus by James Stewart.

www.bookspar.com | VTU NOTES | QUESTION PAPERS 86 of 124


www.bookspar.com | VTU NOTES | QUESTION PAPERS | NEWS | RESULTS | FORUMS

2011
Suppose we have a flat metal plate where the temperature at a point (x,y) varies
according to position. In particular, let the temperature at a point (x,y) be given by,

T ( x, y ) = 60 /1 + x 2 + y 2
where T is measured in oC and x and y in meters.
Question: what is the rate of change of temperature with respect to distance at the point
(2,1) in (a) the x-direction? and (b) in the y-direction ?
Let's take (a) first.
What is the rate of change of temperature with respect to distance at the point (2,1) in
(a) the x-direction?
What observations and translations can we make here?
Rate of change of temperature indicates that we will be computing a type of derivative.
Since the temperature function is defined on two variables we will be computing a partial
derivative. Since the question asks for the rate of change in the x-direction, we will be
holding y constant. Thus, our question now becomes:
What is dT dxat the point (2,1)?
T ( x, y ) = 60 /1 + x 2 + y 2 = 60(1 + x 2 + y 2 ) −1
∂T = −60(2 x)(1 + x 2 + y 2 ) −2
∂x
∂T (2,1) = −60(4)(1 + 4 + 1) −2 = −20
∂x 3
Conclusion :
The rate of change of temperature in the x-direction at (2,1) is −20 3 degrees
per meter;
note this means that the temperature is decreasing !

www.bookspar.com | VTU NOTES | QUESTION PAPERS 87 of 124


www.bookspar.com | VTU NOTES | QUESTION PAPERS | NEWS | RESULTS | FORUMS

2011

Part (b):
The rate of change of temperature in the y-direction at (2,1) is computed in a similar
manner. T ( x, y ) = 60 /1 + x 2 + y 2 = 60(1 + x 2 + y 2 ) −1
∂T = −60(2 y )(1 + x 2 + y 2 ) −2
∂x
∂T (2,1) = −60(2)(1 + 4 + 1) −2 = −10
∂x 3
Conclusion :
The rate of change of temperature in the y-direction at (2,1) is −10 degrees
3
per meter;
note this means that the temperature is decreasing !

II. Electrical Circuits: Changes in Current


The following is adapted from an example in a former text for SM221,223 Multivariable
Calculus by Bradley and Smith.
* In an electrical circuit with electromotive force (EMF) of E volts and resistance R ohms,
the current, I, is
I=E/R amperes.
Question: (a) At the instant when E=120 and R=15 , what is the rate of change of
current with respect to voltage.
(b) What is the rate of change of current with respect to resistance?

(a) Even though no geometry is involved in this example, the rate of change questions
can be answered with partial derivatives.
we first note that I is a function of E and R, namely,
I(E,R) = ER-1

www.bookspar.com | VTU NOTES | QUESTION PAPERS 88 of 124


www.bookspar.com | VTU NOTES | QUESTION PAPERS | NEWS | RESULTS | FORUMS

The rate of change of current with respect to voltage =


the partial derivative of I with respect to voltage, holding resistance constant is
∂I = R −1
∂E

when E=120 and R=15 , we have ∂ I ∂ E = 15 −1 ≈ 0.0667


verbal conclusion : If the resistance is fixed at 15 ohms, the current is increasing with
respect to voltage at the rate of 0.0667 amperes per volt when the EMF is 120 volts.
Part (b):
What is the rate of change of current with respect to resistance?
Using similar observations to part (a) we conclude:

The partial derivative of I with respect to resistance, holding voltage constant = ∂I ∂E = ER −1


when E=120 and R=15 , we have ∂I ∂E (120,15) = −120(15)−1 ≈ −0.5333

Conclusion : If the EMF is fixed at 120 volts, the current is decreasing with respect to
resistance at the rate of 0.5333 amperes per ohm when the resistance is 15 ohms.

Key Words :-
Then the partial derivative of z w.r.t x is given by
∂z f ( x + δ x, y ) − f ( x, y )
zx = = lim
∂x δ x→0 δx
The partial derivative of z w.r.t y is given by
∂z f ( x, y + δ y ) − f ( x, y )
zy = = lim
∂y δ y →0 δy

www.bookspar.com | VTU NOTES | QUESTION PAPERS 89 of 124


www.bookspar.com | VTU NOTES | QUESTION PAPERS | NEWS | RESULTS | FORUMS

2011
∂u ∂u
1. If u = e ax -by sin (ax + by ) show that b -a = 2 ab u
∂x ∂y

Solution : u = e ax -by sin (ax + by )


∂u
∴ = e ax - by cos (ax + by ) . a + a.e ax -by sin (ax + by )
∂x
∂u
ie., = a e ax - by cos (ax + by ) + au …(1)
∂x
∂u
Also = e ax - by cos (ax + by ) . b + (- b ) e ax -by sin (ax + by )
∂x
∂u
ie., = b e ax - by cos (ax + by ) − bu …(2)
∂y
∂u ∂u
Now b =a by using (1) and (2) becomes
∂x ∂y

= abe ax − by cos (ax + by ) + abu - abe ax -by cos (ax + by ) + abu


= 2 abu
∂u ∂u
Thus b - a = 2 abu
∂x ∂y

2. If u = eax + by f (ax - by ), prove that


∂u ∂u
b =a = 2abu
∂x ∂y

Solution : u = e ax+ by f (ax - by ) , by data


∂u
= e ax + by . f ′ (ax - by ) a + ae ax + by f (ax - by )
∂x
∂u
Or = a e ax + by . f ′ (ax - by ) + a u …(1)
∂x
∂u
Next, = e ax + by f ′ (ax - by ) . (- b ) + b e ax + by f (ax - by )
∂y
∂u
Or = − b e ax + by f ′ (ax - by ) + ba …(2)
∂y
∂u ∂u
Now consider L.H.S = b + a
∂x ∂y

www.bookspar.com | VTU NOTES | QUESTION PAPERS 90 of 124


www.bookspar.com | VTU NOTES | QUESTION PAPERS | NEWS | RESULTS | FORUMS

2011
{ } {
= b ae ax + by f ′ (ax - by ) + au + a - be ax + by f ′ (ax - by ) + bu }
= ab e ax + by f ′ (ax - by ) + abu - ab e ax + by f ′ (ax - by ) + abu
= 2abu = R.H.S
∂u ∂u
Thus b +a = 2a bu
∂x ∂y

⎛ ∂ 2u ∂ 2u ∂ 2u ⎞
3. ( )
If u = log x 2 + y 2 + z 2 , show that x 2 + y 2 + z 2 ⎜⎜ 2 + 2 + 2 ⎟⎟ = 1
⎝ ∂x ∂y ∂z ⎠

Solution : By data u = log x 2 + y 2 + z 2 = log (x 2 + y 2 + z 2 )


1
2
The given u is a symmetric function of x, y, z,
(It is enough if we compute only one of the required partial derivative)
∂u 1 1 x
= . 2 . 2x = 2
∂x 2 x + y + z
2 2
x + y2 + z 2

∂ 2 u ∂ ⎛ ∂u ⎞ ∂ ⎛ x ⎞
= ⎜ ⎟= ⎜⎜ 2 ⎟
2 ⎟
∂x 2
∂x ⎝ ∂x ⎠ ∂x ⎝ x + y + z ⎠
2

ie.,
(x 2
)
+ y2 + z2 1 - x . 2 x
=
y2 + z2 − x2
(x 2
+ y2 + z2 )
2
(x 2
+ y2 + z2 )
2

∂ 2u y2 + z2 − x2
∴ = …(1)
∂x 2 (
x2 + y2 + z2
2
)
∂ 2u z2 + x 2 − y2
Similarly ∴ = …(2)
∂y 2 (
x2 + y2 + z2
2
)
∂ 2u x 2 + y2 − z 2
= …(3)
∂z 2 (
x2 + y2 + z2
2
)
Adding (1), (2) and (3) we get,
∂ 2u ∂ 2u ∂ 2u x 2 + y2 + z2 1
+ + = = 2
∂x 2
∂y 2
∂z 2
x2 + y2 + z2 (
2
x + y2 + z 2 )
( )
⎛ ∂ 2u ∂ 2u ∂ 2u ⎞
Thus x 2 + y 2 + z 2 ⎜⎜ 2 + 2 + 2 ⎟⎟ = 1
⎝ ∂x ∂y ∂z ⎠

www.bookspar.com | VTU NOTES | QUESTION PAPERS 91 of 124


www.bookspar.com | VTU NOTES | QUESTION PAPERS | NEWS | RESULTS | FORUMS

2011
4. If u = log (tan x + tan y + tan z), show that,
sin 2x ux + sin 2y uy + sin 2z uz = 2
Solution : u = log (tan x + tan y + tan z) is a symmetric function.
sec 2 x
ux =
tan x + tan y + tan z

(2 sin x cos x ) sec2 . x


sin 2 x u x =
tan x + tan y + tan z
2 tan x
Or sin 2 x u x = …(1)
tan x + tan y + tan z
2 tan y
Similarly sin 2 y u y = …(2)
tan x + tan y + tan z
2 tan z
sin 2 z u y = …(3)
tan x + tan y + tan z
Adding (1), (2) and (3) we get,
2 (tan x + tan y + tan z )
sin 2 x u x + sin 2y u y + sin 2z u z = =2
(tan x + tan y + tan z )
Thus sin 2x u x + sin 2y u y + sin 2z u z = 2

∂u ∂u ∂u 3
5. If u = log (x3 + y3 + z3 – 3xyz) then prove that + + = and hence show
∂x ∂y ∂ z x + y + z
2
⎛∂ ∂ ∂⎞ -9
that ⎜⎜ + + ⎟⎟ u =
⎝ ∂x ∂y ∂z ⎠ (x + y + z )2
Solution : u = log (x3 + y3 + z3 – 3xyz) is a symmetric function
∂u 3x 2 - 3yz
= 3 …(1)
∂x x + y3 + z 3 - 3xyz

∂u 3 y 2 - 3zx
= 3 …(2)
∂y x + y3 + z 3 - 3xyz

∂u 3z 2 - 3xy
= 3 …(3)
∂z x + y3 + z 3 - 3xyz
Adding (1), (2) and (3) we get,

www.bookspar.com | VTU NOTES | QUESTION PAPERS 92 of 124


www.bookspar.com | VTU NOTES | QUESTION PAPERS | NEWS | RESULTS | FORUMS

2011

+ + =
(
∂u ∂u ∂u 3 x 2 + y 2 + z 2 − xy − yz − zx )
∂x ∂y ∂z (
x 3 + y3 + z 3 − 3xyz )
Recalling a standard elementary result,
(
a 3 + b 3 + c 3 − 3abc = (a + b + c ) a 2 + b 2 + c 2 − ab − bc − ca )
We have,
∂ u ∂ u ∂u
+ + =
(
3 x 2 + y 2 + z 2 − xy − yz − zx )
(
∂x ∂y ∂z ( x + y + z ) x 2 + y 2 + z 2 − xy − yz − zx )
∂u ∂u ∂ u 3
Thus + + =
∂ x ∂y ∂z x + y + z
2
⎛ ∂ ∂ ∂ ⎞
Further ⎜⎜ + + ⎟⎟ u
⎝ ∂x ∂y ∂z ⎠
⎛ ∂ ∂ ∂ ⎞ ⎛ ∂ ∂ ∂ ⎞
= ⎜⎜ + + ⎟⎟ ⎜⎜ + + ⎟⎟ u
⎝ ∂x ∂y ∂z ⎠ ⎝ ∂x ∂y ∂z ⎠
⎛ ∂ ∂ ∂ ⎞ ⎛ ∂u ∂u ∂u ⎞
= ⎜⎜ + + ⎟⎟ ⎜⎜ + + ⎟⎟
⎝ ∂x ∂ y ∂ z ⎠ ⎝ ∂x ∂y ∂z ⎠
⎛ ∂ ∂ ∂ ⎞ ⎛ 3 ⎞
= ⎜⎜ + + ⎟⎟ ⎜⎜ ⎟⎟ , by using the earlier result.
⎝ ∂x ∂ y ∂ z ⎠ ⎝ x+ y+ z⎠

∂ ⎛ 3 ⎞ ∂ ⎛ 3 ⎞ ∂ ⎛ 3 ⎞
= ⎜⎜ ⎟⎟ + ⎜⎜ ⎟⎟ + ⎜⎜ ⎟
∂x ⎝ x + y + z ⎠ ∂y ⎝ x + y + z ⎠ ∂z ⎝ x + y + z ⎟⎠
−3 −3 −3 −9
= + + =
(x + y + z) 2
(x + y + z)
2
(x + y + z) 2
( x + y + z )2
2
⎛ ∂ ∂ ∂ ⎞ -9
Thus ⎜⎜ + + ⎟⎟ u =
⎝ ∂x ∂y ∂z ⎠ ( x + y + z )2

www.bookspar.com | VTU NOTES | QUESTION PAPERS 93 of 124


www.bookspar.com | VTU NOTES | QUESTION PAPERS | NEWS | RESULTS | FORUMS

2011

6. If u = f (r ) and x = r cos θ, y = r sin θ ,

∂ 2u ∂ 2u
prove that 2 + 2 = f ′′ (r ) + f ′ (r )
1
∂x ∂y r
Solution :Observing the required partial derivative we conclude that u must be a function of x, y. But
u = f( r) by data and hence we need to have r as a function of x, y. Since x = r cos θ, y = r
sin θ we have x2 + y2 = r2.

∴ we have u = f (r ) where r = x 2 + y2

∂ 2 u f ′ (r ) 2 2 f ′′ (r )
∂x 2
r
(
= 3 r - x + 2 . x 2 and
r
)
∂ 2 u f ′(r ) 2 2 f ′′ (r ) 2
∂y 2
=
r3
r - y (
+
r2
.y)
Adding these results we get,
∂ 2 u ∂ 2 u f ′ (r ) f ′′(r )
∂x 2
+
∂y 2
{ (
= 3 2x2 - x2 + y2 + 2 x2 + y2
r r
)} ( )
f ′(r ) 2 f ′′(r ) 2 1
= . r + 2 . r = f ′ (r ) + f ′′ (r )
r3 r r
∂ 2u ∂ 2u
+ 2 = f ′′ (r ) + f ′ (r )
1
Thus
∂x 2
∂y r
∂u ∂u
7. Prove that x +y = nu
∂x ∂y

Proof : Since u = f (x, y) is a homogeneous function of degree n we have by the definition,


u = x n g ( y/x ) …(1)
Let us differentiate this w.r.t x and also w.r.t.y
∂u ⎛ y ⎞
∴ = x n . g′ ( y/x ) . ⎜ - 2 ⎟ + nx n - 1 g ( y/x )
∂x ⎝ x ⎠
∂u
ie., = x n - 2 y g′ ( y/x ) + nx n - 1 g ( y / x ) …(2)
∂x
∂u ⎛1⎞
Also = x n . g′ ( y/x ) . ⎜ ⎟
∂y ⎝ x⎠

www.bookspar.com | VTU NOTES | QUESTION PAPERS 94 of 124


www.bookspar.com | VTU NOTES | QUESTION PAPERS | NEWS | RESULTS | FORUMS

2011
∂u
ie., = x n - 1 . g′ ( y/x ) …(3)
∂y
∂u ∂u
Now consider x +y as a consequence of (2) and (3)
∂x ∂y

[ ] [ ]
= x - x n - 2 y g′ ( y/x ) + n x n - 1 g ( y/x ) + y x n - 1 g′ ( y / x )

= − x n −1 y g′ ( y/x ) + n x n g ( y/x ) + x n - 1 y g′ ( y/x )

= n . x n g ( y/x )
= n u, by using (1)
Thus we have proved Euler’s theorem
∂u ∂u
x +y = n u ; x u x + yu y = n u
∂x ∂y

∂ 2u ∂ 2u
8. Prove that x 2 + 2 x y = n (n - 1) u
∂x 2 ∂x ∂y
Proof : Since u = f (x, y) is a homogeneous function of degree n, we have Euler’s theorem
∂u ∂u
x +y =nu …(1)
∂x ∂y
Differentiating (1) partially w.r.t. x and also w.r.t y we get,
⎛ ∂ 2u ∂u ⎞ ∂ 2u ∂u
⎜ x 2 + 1. ⎟+ y = n …(2)
⎜ ∂x ∂x ⎟⎠ ∂x ∂ y ∂x

∂ 2u ⎛ ∂ 2u ∂u ⎞ ∂u
Also, x + ⎜⎜ y 2 + 1 . ⎟=n
⎟ …(3)
∂y ∂x ⎝ ∂y ∂y ⎠ ∂y

We shall now multiply (2) by x and (3) by y.


∂ 2u ∂u ∂ 2u ∂u
x2 + x + x y =nx and
∂x 2
∂x ∂x∂y ∂x

∂ 2u ∂ 2u ∂u ∂u
xy + y2 2 + y = ny
∂y ∂x ∂y ∂y ∂y

∂ 2u ∂ 2u
Adding these using the fact that = we get,
∂y ∂x ∂y ∂x

www.bookspar.com | VTU NOTES | QUESTION PAPERS 95 of 124


www.bookspar.com | VTU NOTES | QUESTION PAPERS | NEWS | RESULTS | FORUMS

2011
⎛ 2 ∂ 2u ∂ 2u 2 ∂ u ⎞ ⎛ ∂u
2
∂u ⎞ ⎛ ∂u ∂u ⎞
⎜x + 2 xy + y ⎟ + ⎜⎜ x + y ⎟ = n ⎜ x + y ⎟
⎜ ∂x

2
∂x∂y ∂y ⎟⎠ ⎝ ∂x
2 ⎟
∂y ⎠ ⎜
⎝ ∂x ∂y ⎟⎠

∂ 2u ∂ 2u 2 ∂ u
2
ie., x 2 + 2 x y + y + n u = n (n u ), by using (1)
∂x 2 ∂x∂y ∂y 2

∂ 2u ∂ 2u 2 ∂ u
2
or x 2
+2xy +y + n (nu ) - nu = n (n - 1) u
∂x 2 ∂x∂y ∂y 2

∂ 2u ∂ 2u 2 ∂ u
2
Thus x 2
+2xy +y + n (n - 1) u
∂x 2 ∂x∂y ∂y 2

ie., x 2 u xx + 2 x y u xy + y 2 u yy = n (n - 1) u

x y z ∂u ∂u ∂u
9. If u = + + show that x +y +z =0
y+z z+ x x+y ∂x ∂y ∂z
Solution : (Observe that the degree is 0 in every term)
x y z
u= + +
y+ z z+ x x+ y
We shall divide both numerator and denominator of every term by x.

= x {g ( y/x, z/x )}
1 y/ x z
u= + +
y/x + z/x z / x + 1 1 + y / x
⇒ u is homogeneous of degree 0. ∴ n = 0
∂u ∂u ∂u
We have Euler’s theorem, x +y +z =nu
∂x ∂y ∂z
∂u ∂u ∂u
Putting n = 0 we get, x +y +z =0
∂x ∂y ∂z

⎛ x4 + y4 ⎞ ∂u ∂u
10. If u = log ⎜⎜ ⎟⎟ show that x +y =3
⎝ x+ y ⎠ ∂x ∂y

Solution : we cannot put the given u in the form xn g (y/x)

∴e =
(
x4 + y4 x4 1 + y4 / x4
u
= =x ⎨
)
3 ⎪1 + ( y/x ) ⎪
⎧ 4⎫

x+ y x (1 + y/x ) ⎪⎩ 1 + ( y / x ) ⎪⎭

ie., eu = x3 g (y/x) ⇒ eu is homogeneous of degree 3 ∴ n = 3


Now applying Euler’s theorem for the homogeneous function eu

www.bookspar.com | VTU NOTES | QUESTION PAPERS 96 of 124


www.bookspar.com | VTU NOTES | QUESTION PAPERS | NEWS | RESULTS | FORUMS

2011

We have x
( )
∂ eu
+y
∂ eu( )
= n eu
∂x ∂y
∂u ∂u
ie., x e u + y eu = 3e u
∂x ∂y
∂u ∂u
Dividing by eu we get x +y =3
∂x ∂y

⎛ x3 + y3 ⎞
11. If u = tan -1 ⎜⎜ ⎟⎟ show that
⎝ x− y ⎠
(i) x ux + y uy = sin 2 u
(ii) x2uxx + 2 x y uxy + y2uyy = sin 4 u – sin 2 u
⎛ x 3 + y3 ⎞
Solution : (i) u = tan ⎜⎜ -1 ⎟ by data

⎝ x - y ⎠

⇒ tan u = =
(
x3 + y3 x3 1 + y3 / x3 ) ⎧⎪1 + ( y/x )3 ⎫⎪
= x2 ⎨ ⎬
x-y x (1 - y/x ) ⎪⎩1 − ( y / x ) ⎪⎭

ie., tan u = x2/g (y/x) ⇒ tan u is homogeneous of degree 2.


Applying Euler’s theorem for the function tan u we have,
∂ (tan u ) ∂ (tan u )
x +y = n . tan u ; n = 2
∂x ∂y
∂u ∂u
ie., x sec 2 u + y sec 2 u = 2 tan u
∂x ∂y
∂u ∂u 2 tan u sin u
or x +y = = 2 cos 2 u = 2 cos u sin u = sin 2 u
∂x ∂y 2
sec u cos u
∴ xu x + yu y = sin 2u

(ii) We have xux + y uy = sin 2 u …(1)


Differentiating (1) w.r.t x and also w.r.t y partially we get
x u xx + 1 . u x + yu xy = 2 cos 2u . u x …(2)

And x uyx + yuyy + 1 . uy = 2 cos 2u . uy …(3)


Multiplying (2) by x and (3) by y we get,
x 2 u xx + x u x + xy u xy = 2 cos 2 u. xu x

www.bookspar.com | VTU NOTES | QUESTION PAPERS 97 of 124


www.bookspar.com | VTU NOTES | QUESTION PAPERS | NEWS | RESULTS | FORUMS

2011
xy u yx + y 2 u yy + y u y = 2 cos 2 u. yu y

Adding these by using the fact that uyx = uxy, we get


( ) (
x 2 u xx + 2 x y u xy + y 2 u yy + xy x + yu y = 2 cos 2u xu x + y u y )
By using (1) we have,
x 2 u xx + 2 x y u xy + y 2 u yy = 2 cos 2u sin 2 u - sin 2u

(since sin 2θ = 2 cos θ sin θ, first term in the R.H.S becomes sin 4u)
Thus x2uxx + 2 x y uxy + y2 uyy = sin 4 u – sin 2u
⎛x y z⎞ ∂u ∂u ∂u
12. If u = f ⎜⎜ , , ⎟⎟ Prove that x +y +z =0
⎝y z x⎠ ∂x ∂y ∂z
>> here we need to convert the given function u into a composite function.

Let u = f (p, q, r ) where p =


x y z
,q= ,r =
y z x
ie., {u → (p, q, r ) → (x, y, z )} ⇒ u → x, y, z
∂u ∂ u ∂p ∂ u ∂ q ∂u ∂ r
∴ = + +
∂x ∂ p ∂x ∂q ∂x ∂r ∂x
∂u ∂u 1 ∂u ∂u ⎛ z ⎞
ie., = . + . 0+ .⎜- ⎟
∂x ∂p y ∂q ∂r ⎝ x 2 ⎠
∂u x ∂u z ∂u
∴ x = - …(1)
∂x y ∂p x ∂r
Similarly by symmetry we can write,
∂u y ∂u x ∂u
y = - …(2)
∂ y z ∂ q y ∂p
∂ u z ∂ u y ∂u
z = - …(3)
∂z x ∂r z ∂q
∂u ∂u ∂u
Adding (1), (2) and (3) we get x +y +z =0
∂x ∂y ∂z

www.bookspar.com | VTU NOTES | QUESTION PAPERS 98 of 124


www.bookspar.com | VTU NOTES | QUESTION PAPERS | NEWS | RESULTS | FORUMS

2011
∂u ∂u ∂u
13. If u = f(x – y, y – z, z – x) show that + + =0
∂x ∂y ∂z
>> Let u = f(p, q, r) where p = x – y, q = y – z, r = z – x
∂ u ∂ u ∂p ∂u ∂ q ∂u ∂r
∴ = + +
∂ x ∂ p ∂x ∂ q ∂x ∂ r ∂ x
∂u ∂u ∂u ∂u
ie., = .1+ .0 + (- 1)
∂x ∂p ∂q ∂r
∂u ∂u ∂ u
∴ = - …(1)
∂ x ∂p ∂ r
Similarly we have by symmetry
∂u ∂u ∂u
= - …(2)
∂ y ∂ q ∂p
∂ u ∂u ∂u
= - …(3)
∂z ∂r ∂ q
∂ u ∂u ∂u
Adding (1), (2) and (3) we get, + + =0
∂ x ∂y ∂z

14. If z = f(x, y) where x = r cos θ and y = r sin θ


2
⎛ ∂z ⎞ ⎛ ∂z ⎞ ⎛ ∂z ⎞
2 2 2
1 ⎛ ∂z ⎞
Show that ⎜ ⎟ + ⎜⎜ ⎟⎟ = ⎜ ⎟ + 2 ⎜ ⎟
⎝ ∂ x ⎠ ⎝ ∂y ⎠ ⎝ ∂ r ⎠ r ⎝ ∂θ ⎠
Solution : {z → ( x, y ) → (r, θ ) } ⇒ z → (r, θ )
∂z ∂z ∂ x ∂z ∂y ∂ z ∂x ∂z ∂ y
∴ = + ; +
∂ r ∂x ∂r ∂ y ∂ r ∂ x ∂θ ∂ y ∂θ
∂z ∂ z ∂z
ie., = cos θ + sin θ …(1)
∂r ∂x ∂y

∂z ∂ z ⎡ ⎤
and = ( - r sin θ) + ∂z (r cos θ) = r ⎢ - ∂z sin θ + ∂z cos θ⎥
∂θ ∂ x ∂y ⎣ ∂x ∂y ⎦
1 ∂z − ∂z ∂z
or = = sin θ + cos θ
r ∂θ ∂x ∂y
squaring and adding (1), (2) and collecting suitable terms have,

www.bookspar.com | VTU NOTES | QUESTION PAPERS 99 of 124


www.bookspar.com | VTU NOTES | QUESTION PAPERS | NEWS | RESULTS | FORUMS

2011

[ ]
2 2 2
⎛ ∂z ⎞ 1 ⎛ ∂z ⎞ ⎛ ∂ z ⎞
⎜ ⎟ + 2 ⎜ ⎟ = ⎜ ⎟ cos θ + sin θ
2 2
⎝ ∂r ⎠ r ⎝ ∂θ ⎠ ⎝ ∂x ⎠

[ ]
2
⎛ ∂z ⎞ ∂z ∂z ∂z ∂z
+ ⎜⎜ ⎟⎟ sin 2 θ + cos 2 θ + 2 cos θ sin θ - 2 sin θ cos θ
⎝ ∂y ⎠ ∂x ∂ y ∂x ∂y
2
1 ⎛ ∂z ⎞ ⎛ ∂z ⎞ ⎛ ∂z ⎞
2 2 2
⎛ ∂z ⎞
∴ ⎜ ⎟ ⎜ ⎟ = ⎜ ⎟ + ⎜ ⎟ ie., R.H.S = L.H.S
⎝ ∂r ⎠ r 2 ⎝ ∂θ ⎠ ⎝ ∂x ⎠ ⎜⎝ ∂y ⎟⎠

15. If z = f (x, y ) where x = e u + e − v , y = e -u − e v


∂z ∂z ∂z ∂z
Prove that x −y = −
∂x ∂y ∂u ∂v
Solution : {z → (x, y ) → (u, v )} ⇒ z → (u, v )
∂ z ∂ z ∂ x ∂ z ∂ y ∂ z ∂ z ∂x ∂ z ∂ y
∴ = + ; = +
∂ u ∂ x ∂ u ∂y ∂u ∂ v ∂ x ∂ v ∂ y ∂ v
∂z ∂z ∂z
ie., =
∂u ∂x
. eu +
∂y
- e -u ( ) …(1)

=
∂ v ∂x
( )
∂ z ∂z - v ∂ z
-e +
∂y
- e-v ( ) …(2)

∂ z ∂z
Consider R.H.S = − and (1) – (2) yields
∂ u ∂v
∂z u
∂x
(
e + e-v -
∂z - u
∂y
)
e − ev =
∂z
∂x
.x-(∂z
∂y
. y )
∂z ∂z ∂z ∂z
Thus - =x −y ie., R.H.S = L.H.S
∂u ∂y ∂x ∂y
∂ (u , v, w )
16. Find where u = x2 + y2 + z2, v = xy+yz+zx, w=x+y+z
∂ (x , y, z )

∂u ∂u ∂u
∂x ∂y ∂z
∂ (u , v, w ) ∂v ∂v ∂v
Solution : The definition of J = =
∂ (x , y, z ) ∂x ∂y ∂z
∂w ∂w ∂w
∂x ∂y ∂z

But u = x2 + y2 + z2, v = xy+yz+zx, w=x+y+z

www.bookspar.com | VTU NOTES | QUESTION PAPERS 100 of 124


www.bookspar.com | VTU NOTES | QUESTION PAPERS | NEWS | RESULTS | FORUMS

2011

Substituting for the partial derivatives we get

2x 2y 2z
J= y+z x+z y+x
1 1 1

Expanding by the first row,


J = 2x {(x + z) – (y + x)} -2y {(y + z) – (y + x)}
+2z {(y + z) – (x + z)}
= 2x (z-y) – 2y(z-x) + 2z(y-x)
= 2(xz – xy – yz + xy + yz – xz) = 0 Thus J = 0
yz zx xy ∂ (u , v, w )
17. If u = ,v= ,w= , show that =4
x y z ∂ (x , y, z )

yz zx xy
Solution : by data u = ,v= ,w=
x y z

∂u ∂u ∂u − yz z y
∂x ∂y ∂z x2 x x
∂ (u , v, w ) ∂v ∂v ∂v z − zx x
= =
∂ (x , y, z ) ∂x ∂y ∂z y y2 y
∂w ∂w ∂w y x − xy
∂x ∂y ∂z z z z2

− yz ⎧⎛ − zx ⎞ ⎛ − xy ⎞ ⎛ x ⎞ ⎛ x ⎞⎫
= ⎨⎜⎜ 2 ⎟⎟ ⎜ 2 ⎟ − ⎜ ⎟ ⎜⎜ ⎟⎟⎬
x2 ⎩⎝ y ⎠ ⎝ z ⎠ ⎝ z ⎠ ⎝ y ⎠⎭

z ⎧ z ⎛ − zx ⎞ y x ⎫ y ⎧ z x y ⎛ − zx ⎞⎫
- ⎨ ⎜ ⎟− ⋅ ⎬+ ⎨ ⋅ − ⎜ ⎟⎬
x ⎩ y ⎜⎝ y 2 ⎟⎠ z y ⎭ x ⎩ y z z ⎜⎝ y 2 ⎟⎠⎭

− yz ⎧ x 2 x 2 ⎫ z ⎧ − x x ⎫ y ⎧ x x⎫
= 2 ⎨ − ⎬− ⎨ − ⎬+ ⎨ + ⎬
x ⎩ yz yz ⎭ x ⎩ z z ⎭ x ⎩ y y⎭

= 0+1+1+1+1=4

∂ (u , v, w )
Thus =4
∂ (x , y, z )

www.bookspar.com | VTU NOTES | QUESTION PAPERS 101 of 124


www.bookspar.com | VTU NOTES | QUESTION PAPERS | NEWS | RESULTS | FORUMS

2011
18. If u + v = ex cos y and u – v = ex sin y find the jacobian of the functions u and v w.r.t x
and y.

∂u ∂u
∂ (u , v ) ∂x ∂y
Solution : we have to find =
∂ (x , y ) ∂v ∂v
∂x ∂y

Using the given data we have to solve for u and v in terms of x and y.

By data u + v = ex cos y ……(1)


u – v = ex sin y ……(2)

(1) + (2) gives : 2 u = ex (cos y + sin y)


(2) – (2) gives : 2 v = ex (cos y – sin y)
ex ex
Ie., u = (cos y + sin y) ; v = (cos y – sin y)
2 2
∂u ex ∂v e x
∴ = (cos y + sin y), = (- sin y - cos y)
∂x 2 ∂x 2

ex ex
∂ (u , v ) (cos y + sin y) ( − sin y + cos y)
Now = 2x 2
∂ ( x, y ) e − ex
(cos y − sin y) (sin y + cos y)
2 2
ex ex
= . { - ( cos y + sin y)2 – (cos y – sin y)2}
2 2
− e2x − e2x
= {1+sin 2y) + (1 – sin 2y)} =
4 2
∂ (u , v ) − e 2 x
Thus =
∂ (x , y ) 2

www.bookspar.com | VTU NOTES | QUESTION PAPERS 102 of 124


www.bookspar.com | VTU NOTES | QUESTION PAPERS | NEWS | RESULTS | FORUMS

2011

∂ (r, θ)
19. (a) If x = r cos θ , y = r sin θ find the value of
∂ (x , y )

∂ (x , y ) ∂ (r, θ)
(b) Further verify that . =1
∂ (r, θ) ∂ (x , y )

(a) Solution : We shall first express r, θ in terms of x and y.


We have x = r cos θ , y = r sin θ by data.

y
∴ x2 + y2 = r2 and = tan θ or θ = tan-1 (y/x)
x
Consider r2 = x2 + y2
Differentiating partially w.r.t x and also w.r.t y we get,
∂r ∂r
2r = 2x and 2r = 2y
∂x ∂y
∂r x ∂r y
∴ = and =
∂x r ∂y r

Also consider θ = tan −1 ( y / x )


∂θ 1 −y ∂θ 1 1
∴ = . 2 and = .
∂ x 1 + (y / x) 2
x ∂ y 1 + (y / x) 2
x

∂θ −y ∂θ x
i.e., = 2 and = 2
∂ x x + y2 ∂ y x + y2

∂r ∂r x y
∂ (r ,θ ) ∂r dy r r
Now = =
∂ ( x, y ) ∂θ ∂θ −y x
dx dy x + y2
2
x + y2
2

x2 y2 (x2 + y 2 ) 1
i.e., = + = =
r(x 2 + y 2 ) r(x 2 + y 2 ) r(x 2 + y 2 ) r
∂ (r , θ ) 1
∴ =
∂ ( x, y ) r

www.bookspar.com | VTU NOTES | QUESTION PAPERS 103 of 124


www.bookspar.com | VTU NOTES | QUESTION PAPERS | NEWS | RESULTS | FORUMS

Solution (b) : Consider x = r cos θ , y = r sin θ


∂x ∂x
∂ ( x, y ) ∂r ∂θ cosθ − r sin θ
= = = r (cos 2 θ + sin 2 θ ) = r
∂(r , θ ) ∂y ∂y sin θ r cosθ
∂r dθ
∂ ( x, y )
∴ =r
∂(r , θ )

∂ ( x, y ) ∂ (r ,θ ) 1
From (1) and (2) : ⋅ = r ⋅ =1
∂ ( x,θ ) ∂ ( x, y ) r

20. If x = u (1 − v ), y = uv then show that JJ / = 1

Solution : x = u (1 − v); y = uv
∂x ∂y ∂x ∂y
= (1 − v), =v = −u , =u
∂u ∂u ∂v ∂v

∂x ∂x
∂ ( x, y ) ∂u ∂v (1 − v) − u
J= = =
∂(u, v) ∂y ∂y v u
∂u ∂v
= (1 − v)u + uv = u ∴ J =u
Next we shall express u and v in terms of x and y.
By data x = u - uv and y = uv

y y
Hence x + y = u. Also v = =
u x+ y
y ∂u ∂u
Now we have, u = x + y; v = ∴ = 1, = 1,
x+ y ∂x ∂y

www.bookspar.com | VTU NOTES | QUESTION PAPERS 104 of 124


www.bookspar.com | VTU NOTES | QUESTION PAPERS | NEWS | RESULTS | FORUMS

2011
∂v ( x + y ) ⋅ 0 − y ⋅ 1 x
= =
∂x ( x + y)2 ( x + y) 2

∂u ∂u
1 1
∂ (u , v) ∂x ∂y
∴J/ = = = −y x
∂ ( x, y ) ∂v ∂v
( x + y) 2 ( x + y) 2
∂x ∂y
x y x+ y 1 1
= + = = =
( x + y) 2
( x + y) 2
( x + y) 2
( x + y) u
1 1
Thus J / = Hence J ⋅ J / = u ⋅ Thus JJ / = 1
u u

21. State Taylor’s Theorem for Functions of Two Variables.

Statement: Considering f (x + h, y + k) as a function of a single variable x, we have by


Taylor’s Theorem

∂f ( x, y + k ) h 2 ∂ 2 f ( x, y + k )
f (x + h, y + k) = f (x, y + k) + h + + ---------(1)
∂x 2! ∂x 2

Now expanding f (x, y + k) as function of y only,

∂f ( x, y ) k 2 ∂ 2 f ( x, y )
f (x, y + k) = f (x, y) + k + + --------
∂y 2! ∂y 2

∂f ( x, y ) k 2 ∂ 2 f ( x, y )
∴ (i) takes the form f (x + h, y + k) = f (x,y) + k + +-------------------------------- +
∂y 2! ∂y 2
∂ ⎧ ∂f ( x, y ) k 2 ∂ 2 f ( x, y ) ⎫
h ⎨ f ( x, y ) + k + + − − − − −⎬
∂x ⎩ ∂y 2! ∂y 2

h2 ∂2 ⎧ ∂ ( x, y ) ⎫
+ ⎨ f ( x , y ) + k + − − − − − − − − ⎬
2! ∂x 2 ⎩ ∂y ⎭

∂f ∂f 1 ⎛ 2 ∂ 2 f ∂2 f 2 ∂ f
2

Hence f (x + h, y + k) = f (x , y) +h +k + ⎜⎜ h + 2 hk + k ⎟⎟ + − − (1)
∂x ∂y 2! ⎝ ∂x 2 ∂x∂y ∂y 2 ⎠

In symbol we write it as

www.bookspar.com | VTU NOTES | QUESTION PAPERS 105 of 124


www.bookspar.com | VTU NOTES | QUESTION PAPERS | NEWS | RESULTS | FORUMS

2
⎛ ∂ ∂⎞ 1⎛ ∂ ∂⎞
F (x + h, y + k) = f (x,y) + ⎜⎜ h + k ⎟⎟ f + ⎜⎜ h + k ⎟⎟ f + − − − −
⎝ ∂x ∂y ⎠ 2! ⎝ ∂x ∂y ⎠

Taking x = a and y = b, (1) becomes

1
f (a + h, b + k) = f (a,b) + [h f x (a,b) + kf y (a,b)] + [h 2 f xx (a,b)+ 2hkf xy (a,b)
2!

+ k 2 f yy (a,b)] + ---------

Putting a + h = x and b + k = y so that h = x – a, k = y – b, we get

F (x,y) = f (a,b) + [(x – a) f x (a,b) + (y – b) f y (a,b)]

1
= [(x – a)2 f xx (a,b) + 2 (x – a) (y – b) f xy (a,b) + (y – b)2 f yy (a,b)] +------ (2)
2!

This is Taylor’s expansion of f (x,y) in powers of (x – a) and (y – b). It is used to expand f (x,y)
in the neighborhood of (a,b)

corollary, putting a = 0, b = 0 in (2), we get

1
f (x,y) = f (0,0) + [x f x (0,0) + y f y (0,0)] + [ x 2 f xx (0,0) + 2xy f xy (0,0)
2!
+ y 2 f yy (0,0) ] + --------- (3)
This is Maclaurin’s Expansion of f (x,y)

22. Expand e x log (1 + y) in powers of x and y up to terms of third degree.

Solution: Here

f (x,y) = e x log (1 + y) ∴ f (0,0) = 0

f x (x,y) = e x log (1 + y) ∴ f x (0,0) = 0

1
f y (x,y) = e x ∴ f y (0,0) = 1
1+ y

f xx (x,y) = e x log (1 + y) ∴ f xx (0,0) = 0

1
f xy (x,y) = e x ∴ f xy (0,0) = 1
1+ y

www.bookspar.com | VTU NOTES | QUESTION PAPERS 106 of 124


www.bookspar.com | VTU NOTES | QUESTION PAPERS | NEWS | RESULTS | FORUMS

2011
f yy (x,y) = - e x (1 + y) -2 ∴ f yy (0,0) = -1

f xxx (x,y) = e x log (1 + y) ∴ f xxx (0,0) = 0

1
f xxy (x,y) = e x ∴ f xxy (0,0) = 1
1+ y

f xyy (x,y) = -e x (1 + y) 2 ∴ f xyy (0,0) = -1

f yyy (x,y) = 2e x (1 + y)-3 ∴ f yyy (0,0) = 2

Now, Maclaurin’s expansion of f (x,y) gives


1
f (x,y) = f (0,0) + x (f x (0,0) + y fy (0,0) + {x2 fxx (0,0) + 2xy fxy (0,0) +
2!
1
y2 fyy (0,0)} + {x3 fxxx (0,0) + 3x2 y fxxy (0,0) + 3xy2 fxyy (0,0) + y3 fyyy (0,0)} + -----------
2!

1
∴ex log (1 + y) = 0 + x.0 + y (1) + {x2.0 + 2xy (1) + y2 (-1)}
2!
1 3
+
{x .0 + 3x2y (1) + 3xy2 (-1) + y3(2)}+---------
2!
1 1 1
= y + xy - y2 + (x2y = xy2) + y3 + ----------
2 2 2

⎛ π⎞
23. Expand f (x,y) = ex Cosy by Taylor’s Theorem about the point ⎜1, ⎟ up to the Second
⎝ 4⎠
degree terms.
π ⎛ π⎞
Solution: f (x,y) = ex Cosy and a = 1, b = ∴ f = ⎜1, ⎟ = e
4 ⎝ 4⎠ 2
⎛ π⎞ e
fx (x,y) = ex Cos y ∴ f ⎜1, ⎟ =
⎝ 4⎠ 2
⎛ π⎞ e
fy (x,y) = -ex Sin y ∴ fy ⎜1, ⎟ = -
⎝ 4⎠ 2
⎛ π⎞ e
fxx(x,y) = ex Cos y ∴ fxx ⎜1, ⎟ =
⎝ 4⎠ 2
⎛ π⎞ e
fxy (x,y) = -ex Sin y ∴ fxy ⎜1, ⎟ = -
⎝ 4⎠ 2
⎛ π⎞ e
fyy (x,y) = - ex Cos y ∴ fyy ⎜1, ⎟ = -
⎝ 4⎠ 2
Hence by Taylor’s Theorem, we obtain

www.bookspar.com | VTU NOTES | QUESTION PAPERS 107 of 124


www.bookspar.com | VTU NOTES | QUESTION PAPERS | NEWS | RESULTS | FORUMS

2011

⎛ π⎞ ⎡ ⎛ π⎞ ⎤
f (x,y) = f ⎜1, ⎟ + ⎢( x − 1) f x + ⎜ y − ⎟ fy ⎥ +
⎝ 4⎠ ⎣ ⎝ 4⎠ ⎦
1 ⎡ ⎛ π⎞
2

⎢ x− f x + x − ⎜ y − ⎟ f yy ⎥ + -------------
2
( 1) 2( 1 )
2! ⎣⎢ ⎝ 4⎠ ⎦⎥
e ⎡ e ⎛ π ⎞⎛ e ⎞⎤ 1
i.e., ex Cosy = + ⎢( x − 1) + ⎜ y − ⎟⎜ − ⎟⎥ +
2 ⎣ 2 ⎝ 4 ⎠⎝ 2 ⎠⎦ 2!
⎡ π ⎞⎛ e ⎞ ⎛ π ⎞ ⎛ − e ⎞⎤
2
e ⎛
⎢( x − 1) + 2( x − 1)⎜ y − ⎟⎜ − ⎟+⎜y− ⎟ ⎜ ⎟⎥ + ------------
2

⎣⎢ 2 ⎝ 4 ⎠⎝ 2⎠ ⎝ 4 ⎠ ⎝ 2 ⎠⎦⎥

e ⎡ π ⎞⎤ 1 ⎡ π⎞ ⎛ π⎞ ⎤
2
⎛ ⎛
ex Cosy = ⎢1 + ( x − 1) − ⎜ y − 4 ⎟⎥ + 2! ⎢( x − 1) − 2( x − 1)⎜ y − 4 ⎟ − ⎜ y − 4 ⎟ ⎥ + -------}
2

2 ⎣ ⎝ ⎠⎦ ⎢⎣ ⎝ ⎠ ⎝ ⎠ ⎥⎦

Exercise:

1) Expand exy up to Second degree terms by using Maclaurin’s theorem

2) Expand Log (1 – x – y ) up to Third degree terms by using Maclaurin’s theorem

3) Expand x2y about the point (1,-2) by Taylor’s expansion

4) Obtain the Taylor’s expansion of ex Siny about the point 0, π ( 2


) up to Second degree terms
5) Expand esinx up to the term containing x4

www.bookspar.com | VTU NOTES | QUESTION PAPERS 108 of 124


www.bookspar.com | VTU NOTES | QUESTION PAPERS | NEWS | RESULTS | FORUMS

2011

Maxima and Minima:-


In mathematics, the maximum and minimum (plural: maxima and minima) of a
function, known collectively as extrema (singular: extremum), are the largest and
smallest value that the function takes at a point within a given neighborhood.

A function f (x, y) is said to have a Maximum value at (a,b) if their exists a


neighborhood point of (a,b) (say (a+h, b+k)) such that f (a, b) > f (a+h, b+k).
Similarly,
Minimum value at (a,b) if there exists a neighborhood point of (a,b) (say (a+h,
b+k)) such that f (a, b) < f (a+h, b+k).

A Minimum point on the graph (in red) f ( x, y ) = x 2 + y 2 (1 − x)3

A Maximum point on the graph is at the top (in red)

www.bookspar.com | VTU NOTES | QUESTION PAPERS 109 of 124


www.bookspar.com | VTU NOTES | QUESTION PAPERS | NEWS | RESULTS | FORUMS

2011

ddle point on
A sad n the graph of z=x2−y2 (in red)

Saddle point betw


ween two hillss.

Neccessary an
nd Sufficcient Con
ndition:-

• If fx =0
= and fy =0
= (Necesssary Conddition)
• Functiion will be minimumm if AC-B2 > 0 and A > 0
• Functio
on will be maximum
m if AC-B2 > 0 and A < 0
• Functio
on will be neither
n maaxima nor minima
m if AC-B2 < 0
• If AC C-B2 = 0 we
w cannot make
m any conclusionn without any
a
further anaalysis
w
where A = f xx , B = f xy , C = f yy
A

www.bookspar.com | VTU NOTES | QUESTION PAPERS 110 of 124


www.bookspar.com | VTU NOTES | QUESTION PAPERS | NEWS | RESULTS | FORUMS

2011

Working Procedure:-

• First we find Stationary points by considering


fx =0 and fy =0 .
• Function will be minimum if AC-B2 > 0 and A > 0 at that
stationary point
• Function will be maximum if AC-B2 > 0 and A < 0 at that
stationary point
• Function will be neither maximum nor minimum if AC-B2 < 0
at that stationary point and it is called as SADDLE POINT.

25. Explain Maxima & Minima for Functions of Two Variables& hence obtain the Necessary
Conditions for Maxima, Minima.

Solution: Let Z = f (x,y) be a given function of two independent variables x & y. The above equation
represents a surface in 3D.
Z P(a,b) f (a,b)

Z = f (x,y)

A = f (a,b)
Y
a O
N
b M(a,b)
X

A given points (a,b) on the surface has Co-ordinates [a,b, f (a,b)]

www.bookspar.com | VTU NOTES | QUESTION PAPERS 111 of 124


www.bookspar.com | VTU NOTES | QUESTION PAPERS | NEWS | RESULTS | FORUMS

2011

Definition:
The function Z = f (x,y) is said to be a maximum at the point (a,b) if f (x,y) < f (a,b)
in the neighborhood of the point (a,b)

Cap
Z

Cup

O Y

(a,b)

X
Definition:

The function Z = f (x,y) is said to posses a minimum at the point (a,b) if f (x,y) > f
(a,b) in the neighborhood of the point (a,b)

Necessary Condition for Maxima, Minima:

If Z = f (x,y) has a max or min at (a,b) then f x (a,b) = 0, fy (a,b) = 0

Sufficient Conditions for Maxima, Minima:

Put R = fxx (a,b), S = fxy (a,b), T = fyy (a,b)

(1) Suppose S2 – RT > 0

There is no maxima or minima at (a,b)

(2) Suppose S2 – RT < 0

Thus there is maxima or minima at (a,b) according as R < 0 Or R > 0

(3) Suppose S2 – RT = 0, Then there is a saddle point at (a,b)

www.bookspar.com | VTU NOTES | QUESTION PAPERS 112 of 124


www.bookspar.com | VTU NOTES | QUESTION PAPERS | NEWS | RESULTS | FORUMS

26. Find the maxima and minima of the functions f (x,y) = x3 + y3 – 3axy, a > 0 is constant.

Solution: Given f (x,y) = x3 + y3 – 3axy

fx = 3x2 – 3ay, fy = 3y2 – 3ax

fxx = 6x fyy = 6y.

Put fx = 0, fy = 0 and solve

i.e., 3x2 – 3ay = 0 & 3y2 – 3ax = 0

i.e., x2 = ay & y2 = ax
2
x2 ⎛ x2 ⎞
⇒y= ∴ ⇒ ⎜⎜ ⎟⎟ = ax (∵x2 = ay)
a ⎝ a ⎠

x4
∴ = ax
a2

∴ x4 = a3x

i.e., x (x3 – a3) = 0

∴ x = 0, x = a

⇒ y = 0, y = ± a

∴ The critical Or stationary points are (0,0), (a,a) and (a,-a)

(1) At (0,0)

R = fxx (0,0) = 0

S = fxy (0,0) = -3a

T = fyy (0,0) = 0

∴ S2 – RT = 9a2 – 0 = 9a2 > 0

∴ There is neither a maximum or a minimum at (0,0)

www.bookspar.com | VTU NOTES | QUESTION PAPERS 113 of 124


www.bookspar.com | VTU NOTES | QUESTION PAPERS | NEWS | RESULTS | FORUMS

27. Examine the following functions for extreme values f = x4 + y4 – 2x2 + 4xy – 2y2

Solution:
fx = 4x3 – 4x + 4y

fy = 4y3 – 4x – 4y

fxy = 4, fxx = 12 x2 – 4, fyy = 12y2 – 4

Put fx = 0, fy = 0 and solve

i.e., 4x3 – 4x + 4y = 0 → (1)

4y3 + 4x – 4y = 0 → (2)

Adding (1) & (2), we get

4 (x3 + y3) = 0

i.e., x3 + y3 = 0
i.e., y = - x

Substitute y = -x in (1), we get

4x3 – 4x – 4x = 0

i.e., 4x3 – 8x = 0

i.e., x3 – 2x = 0 ⇒ x (x2 – 2) = 0

i.e., x = 0 & x2 – 2 =0

i.e., x = 0 & x = ± 2

x= 2,- 2

∴x = 0, 2 ,- 2 and corresponding values of y are y = 0, - 2, 2

∴ The critical points are (0,0), 2 ,- 2 , - 2, 2

(1) at (0,0)

R = fxx (0,0) = - 4

www.bookspar.com | VTU NOTES | QUESTION PAPERS 114 of 124


www.bookspar.com | VTU NOTES | QUESTION PAPERS | NEWS | RESULTS | FORUMS

S = fxy (0,0) = 4

T = fyy (0,0) = -4

∴ S2 – RT = 16 – (-4) (-4) = 16 – 16 = 0

i.e., S2 – RT = 0, These is a saddle point at (0,0)

(2) at
2,- 2

R = fxx 2,- 2 =24– 4 = 20

S = fxy 2 ,- 2 =4

T = fyy 2 ,- 2 = 20

∴ S2 – RT = 16 – (20) (20) = 16 – 400 = -384 < 0

Thus these is neither maximum nor minimum according to R < 0 or R > 0 at 2,- 2

Hence R = 20 > 0

∴ There is a minimum at 2 ,- 2

∴ f min = ( 2 ) + (− 2 )
4 4
( )
−2 2
2
( ) (
+4 2 − 2 −2− 2 )
2

=4+4–4–8–4

=-8

(3) at - 2 , 2

(
R = fxx − 2 , 2 = 20 > 0 )
(
S = fxy − 2 , 2 = 4 )
(
T = fyy − 2 , 2 = 20 )
∴ S2 – RT = 16 – 400 = -384 < 0

www.bookspar.com | VTU NOTES | QUESTION PAPERS 115 of 124


www.bookspar.com | VTU NOTES | QUESTION PAPERS | NEWS | RESULTS | FORUMS

(
Since R > 0, ∴ There is minima at − 2 , 2 )
(
∴ fmin = - 8 at − 2 , 2 )
( ) (
∴ Extreme Value = - 8 at − 2 , 2 & − 2 , 2 )
Exercise:

1) Find the extreme values of f = x3 y2 (1 – x – y)


2) Determine the maxima or minima of the function Sin x + Sin y + Sin (x + y)
3) Examine the function f(x,y) = 1+ sin( x2 + y2) for extremum.

28. If PV2 = K and if the relative errors in P is 0.05 and in V is 0.025 show that the error in K
is 10%.
δP δV
Solution : PV 2 = K by data. Also = 0.05 and = 0.025
P V
⇒ log P + 2 log V = log K

⇒ δ (log P ) + 2δ (log V ) = δ (log K )

1 1 1
i.e., δP + 2 ⋅ δV = δK
p V K
δK δK
i.e., 0.05 + 2(0.025) = or = 0.1
K K
δK
∴ × 100 = (0.1) × 100 = 10
K

Thus the error in K is 10%.

www.bookspar.com | VTU NOTES | QUESTION PAPERS 116 of 124


www.bookspar.com | VTU NOTES | QUESTION PAPERS | NEWS | RESULTS | FORUMS

2011
29. The time T of a complete oscillation of a simple pendulum is given by the formula
T = 2π l / g ⋅

(i) If g is a constant find the error in the calculated value of T due to an error of 3%
in the value of l.
(ii) Find the maximum error in T due to possible errors upto 1% in l
and 3% in g.

Solution :

δl
(i ) T = 2π l / g , g = Constant, × 100 = 3
l
1
⇒ log T = log 2π + (log l − log g )
2
1
⇒ δ (log T ) = δ (log 2π ) + δ (log g )
2
δT 1 δl
i.e., = 0+ −0
T 2 l
δT 1 ⎛ δl ⎞ 1
or ×100 = ⎜ × 100 ⎟ = (3) = 1.5
T 2⎝ l ⎠ 2

∴ the error in T = 1.5%.

(ii) If g is not a constant we have,


δT 1 ⎛ δl ⎞ 1 ⎛ δg ⎞
× 100 = ⎜ × 100 ⎟ − ⎜⎜ × 100 ⎟⎟
T 2⎝ l ⎠ 2⎝ g ⎠

The error in T will be maximum if the error in l is positive and the error in g is negative (or vice-
versa) as the difference in errors converts in to a sum.

⎛ δT ⎞ 1 1
∴ max ⎜ × 100 ⎟ = (+1) − (−3) = 2
⎝T ⎠ 2 2
∴ the maximum error in T is 2%.

www.bookspar.com | VTU NOTES | QUESTION PAPERS 117 of 124


www.bookspar.com | VTU NOTES | QUESTION PAPERS | NEWS | RESULTS | FORUMS

2011

30. The current measured by a tangent galvanometer is given by the relation


c = k tan θ where θ is the angle of deflection. Show that the relative error in c due to a
given error in θ is minimum when θ = 45 0.

Solution : Consider c = k tan θ . K is taken as a constant.


⇒ log c = log k + log (tan θ )
⇒ δ (log c) = δ (log k ) + δ log (tan θ )

1 sec 2 θ
i.e., δc = 0 + δθ
c tan θ

δc cos θ 1 δθ δθ
i.e., = ⋅ δθ or =
c sin θ cos 2 θ c sin θ cos θ

δc 2
i.e., = δθ
c sin 2θ

The relative error in c being δc / c minimum when the denominator of the R.H.S. is maximum
and the maximum value of a sine function is1.

∴ sin 2θ = 1 ⇒ 2θ = 90 0 or θ = 45 0 ⋅

Thus the relative error in c is minimum when θ = 450

1
31. If T = mv 2 is the kinetic energy, find approximately the change in T as m changes
2
from 49 to 49.5 and v changes from 1600 to 1590. 6 Marks
1
Solution : We have by data T = mv 2 and
2
m = 49, m + δm = 49.5 ∴ δm = 0.5
v = 1600, v + δv = 1590 ∴ δv = −10

www.bookspar.com | VTU NOTES | QUESTION PAPERS 118 of 124


www.bookspar.com | VTU NOTES | QUESTION PAPERS | NEWS | RESULTS | FORUMS

2011
We have to find δT . (logarithm is not required)
1
∴ δT = δ ( mv 2 )
2
1
2
{
m( 2vδv) + δm.v 2 }
i.e, =
1
2
{
( 49) ( 2) (1600 ) ( −10) + (0.5) (1600 ) 2 = 1,44,000 }
Thus the change in T = δT = −1,44,000

32. The pressure p and the volume v of a gas are concentrated by the relation
pv 1.4 = cons tan t. Find the percentage increase in pressure corresponding to a
diminution of ½% in volume.
Solution :
pv1.4 = Constant = c( say ), by data.
⇒ log p + 1.4 log v = log c
⇒ δ (log p) + 1.4δ (log v) = δ (log c)
δp ⎛ δv ⎞ δv 1
i.e, + 1.4⎜ ⎟ = 0; But × 100 = − , by data.
p ⎝ v⎠ v 2

δp ⎛ δv ⎞ δp
∴ × 100 + 1.4⎜ × 100 ⎟ = 0 or × 100 = +0.7 .
p ⎝ v ⎠ p
Thus the percentage increase in pressure = 0.7

33. Find the percentage error in the area of an ellipse when an error of +1% is made in
measuring the major and minor axis.

Solution : For the ellipse x 2 / a 2 + y 2 / b 2 = 1 the area (A) is given by π ab where 2a and 2b are
the lengths of the major and minor axis.
Let 2a = x and 2b = y.
δx δy
By data × 100 = 1, × 100 = 1.
x y

www.bookspar.com | VTU NOTES | QUESTION PAPERS 119 of 124


www.bookspar.com | VTU NOTES | QUESTION PAPERS | NEWS | RESULTS | FORUMS

x y π
A = πab = π ⋅ ⋅ = xy
2 2 4
∴ log A = log (π / 4) + log x + log y
⇒ δ (log A) = δ log (π / 4) + δ (log x) + δ (log y )
δA δx δy δA δx δy
i.e., =0+ + or × 100 = × 100 + × 100
A x y A x y
δA
∴ × 100 = 1 + 1 = 2
A
Thus error in the area = 2%

27. If the sides and angles of a triangle ABC vary in such way that the circum radius
remains constant, prove that
δa δb δc
+ + =0
cos A cos B cos c
Solution : If the triangle ABC is inscribed in a circle of radius r and if a,b,c respectively
denotes the sides opposite to the angles A,B,C we have the sine rule (formula) given by
a b c
= = = 2r
sin A sin B sin C
or a = 2r sin A, b = 2r sin B, c = 2r sin C
⇒ δa = 2rδ (sin A), δb = 2rδ (sin B), δc = 2rδ (sin C )
i.e., δa = 2r cos AδA, δb = 2r cos BδB, δc = 2r cos CδC
δa δb δc
or = 2rδA, = 2 rδ B , = 2 rδC
cos A cos B cos C
Adding all these results we get,
δa δb δc
+ + = 2r (δA + δB + δC ) = 2rδ ( A + B + C )
cos A cos B cos C

But A+B+C = 180 = π radians = constant.


⇒ δ ( A + B + C ) = δ (constant) = 0
δa δb δc
Thus + + =0
cos A cos B cos C

www.bookspar.com | VTU NOTES | QUESTION PAPERS 120 of 124


www.bookspar.com | VTU NOTES | QUESTION PAPERS | NEWS | RESULTS | FORUMS

2011

Multiple choice Questions:


DIFFERENTIAL CALCULUS

1) The radius of curvature at any point of catenary S=C tan Φ is


a) c sec2 Φ b) c cos2 Φ
c) c tan Φ
-2 d)none

2) Stationary points of f(x,y) = sin x+ sin y+ sin ( x + y ) is


a) (п/3, п/3) b) (п/6, п/6)
c) (п/4, п/4) d) none

3) If the curvature is zero, that point is known as--------------------


a) Point of inflection b) Stationary point
c) (a) or (b) d) none

4) The radius of curvature of the curve y = 4 sin x- sin 2x at x=п/2 is


a) 5√5/4 b) -5√5/4
c)5√5/2 d) none

5) The function x2+2xy+ 2y2+2x+2y has a minimum value at

a) (-3/2,1/2) b)(3/2, 1/2 ) c) (3/2,-1/2) d)none

6) The stationary point of f(x,y,z)=x2+y2+z2 where x,y,z are connected by x+y+z=a is


a) (a,a) b) (-a,-a) c) (2a, 0) d) none

7) The radius of curvature of the curve √x+√y =1 at the point (1/4,1/4) is ------------
a) ρ =1/√2 b) ρ=√2 c) ρ= -1/√2 d) none )

8) The expression for the derivative of arc length in Cartesian form is given by
a) ds/d = r 2 + ( dr/dθ ) 2 b) ds/dx= 1 + (dy/dx) 2
c) ds/dr= 1 + ( d?/dr) 2 d) ds/dt= 1+ (dy/dx)2

9)The formulae for radius of curvature in Cartesian form is

a) ρ = (x1) 2 + (y1) 2)3/2 /x1y11+x11y1

b) ρ =
(1+ y ) 2
1
3/ 2

y2

c) ρ = [(x1) 2 + (y1) 2)3/2 ]

www.bookspar.com | VTU NOTES | QUESTION PAPERS 121 of 124


www.bookspar.com | VTU NOTES | QUESTION PAPERS | NEWS | RESULTS | FORUMS

2011
d) ρ = (1+(y1) 2 ) 3/2

10) The function for which Rolle’s theorem is true is:


a) f(x)=log x in the interval [1/2,2]
b) f(x)=|x+1| in the interval [-2,2]
c) f(x)=| x |in the interval [-1,1]
d) Non of the above

11) The value of ‘c’in Rolle’s theorem ,where − π / 2 < c < π / 2 and f(x)= cosx is equal to:
aπ /4 b) π / 3 c) π d)0

12).The expansion of tanx in powers of x by Maclaurin’s theorem is valid in the interval :


a) (− ∞, ∞ ) b) (− 3π / 2,3π / 2) c) (− π , π ) d) (− π / 2, π / 2)

13) The value of ‘c’in Lagrange,s mean value theorem ,where [1,2]
and f(x)= x(x-1) is :
a)5/4 b)3/2 c)7/4 d) 11/6

14) The value of ‘c’in Rolle’s theorem, where [0, π ]


and f(x)= sinx is equal to:
a) π / 6 b) π / 3 c) π / 2 d) Non of these

15) The maximum value of logx/x is:


a) 1 b)e c)2/e d) 1/e

16) The maximum value of (l/x ) x is equal to:


a)e b)1 c) e 1/e d) (1/e)e

17) The difference bwtween the maximum and minimum values of the function
a sinx+bcosx is:
a) 2 a 2 + b 2 b) 2(a 2 + b 2 ) c) a 2 + b 2 d) − a 2 + b 2

18) Which one of the following statement is correct for the function f(x)=x3
a) f(x) has a maximum value at x=0
b) f(x) has a manimum value at x=0
c) f(x) has a neither a maximum nor a manimum value at x=0
d) f(x) has no point of inflexion

19) which one of the following is not an indeterminate form


a) ∞ + ∞ b) ∞ − ∞ c) ∞ / ∞ d) 0 X∞

20) In Lagrange,s mean value theorem , f1(c)=


a) f(a) – f(b)/ b -a b) f(b) – f(a) / a -b
c) f(b) – f(a) / b -a d) none

www.bookspar.com | VTU NOTES | QUESTION PAPERS 122 of 124


www.bookspar.com | VTU NOTES | QUESTION PAPERS | NEWS | RESULTS | FORUMS

2011

21) In Cauchy’s mean value theorem , f1(c) / g1(c) =


a) f(a) – f(b)/ g(b) – g(a) b) f(b) – f(a) / g(b) – g(a)
.c) f(b) +f(a) / g(b) +g(a) d) none
x x
a - b
22) lim is equal to :
x →0 x

a) 0 b) ∞ c) log (a/b) d) log (a-b)

log x
23) The value of lim is equal to:
x→0 x −1

a) -1 b) ∞ c) 1 d) 0

24) The value of lim (1 – cos x / 3 x2 )is equal to:


x →0

a) 3 b) 1/3 c) 1/6 d) 1/9

25) lim ( ex + e-x – 20) / x2 is equal to :


x→0

a) 1 b) -1 c) 1/2 d) -1/2

26) The value of lim ( 1 + x )1/x is :


x→0

a) 1 b) -1 c) 1/e d) e

sin x
27) The value of lim =….
x →0 x

a) 1 b) 2 c) 3 d)0
28) The formulae for radius of curvature in polar form is

a) ρ = rdr/dp b) ρ = [ (r 2 + (r1) 2)3/2] / [r2+2 ( r1) 2- r r2]

c) ρ =(r 2 + (r1) 2)3/2 d) None

29) The value of lim 1 1/x =……


x →0

a) 1 b) ∞ c) -∞ d) 0

www.bookspar.com | VTU NOTES | QUESTION PAPERS 123 of 124


www.bookspar.com | VTU NOTES | QUESTION PAPERS | NEWS | RESULTS | FORUMS

2011

KEY ANSWERS:

1-a 2-a 3-a 4-a 5-a 6-a 7-a 8-b 9-b 10-a

11-d 12-d 13-b 14-c 15-d 16-c 17-a 18-c 19-a 20-c

21-b 22-c 23-c 24-c 25-a 26-c 27-a 28-b 29-b --

www.bookspar.com | VTU NOTES | QUESTION PAPERS 124 of 124

You might also like